41
People vs. Mariano Facts: The office of the Provincial Fiscal of Bulacan filed an Information accusing Mariano of estafa. Mariano was the Liaison Officer of Mayor Nolasco and is authorized to receive and be receipted for US excess property of USAID/NEC for the use and benefit of the municipality. The property received were electric cables and cable powers amounting to P4,797.35 which he had a duty to deliver to the Mayor. However he willfully, unlawfully and feloniously, with grave abuse of confidence and deceit, misappropriate, misapply and convert to his own personal use and benefit the items. Mariano filed a motion to quash the Information claiming that the court had no jurisdiction. He claimed that the items which were the subject matter of the Information against him were the same items for which Mayor Nolasco was indicted by the Military Commission under a charge of malversation and found guilty. He claimed that inasmuch as the case against Mayor Nolasco has already been decided by the Military Tribunal, the CFI of Bulacan had lost jurisdiction over him. Respondent judge granted the motion to quash stating that since the Military Commission had already taken cognizance of the malversation case involving the same subject matter in its concurrent jurisdiction with the Court, the case for estafa has already been heard and decided. Issue: Whether or not civil courts and military commissions exercise concurrent jurisdiction over estafa and committed by a civilian Held: there is no concurrent jurisdiction Ratio: The question of jurisdiction of respondent CFI is to be resolved on the basis of the law or statute providing for or defining its jurisdiction. The Judiciary Act of 1948 in Section 44 (f) provides the CFI shall have original jurisdiction in all criminal cases in which the penalty provided by law is imprisonment for more than six months or fine of more than 200 pesos. Estafa falls under the original jurisdiction of CFI. Jurisdiction of a court is determined by the statute in force at the time of the commencement of the action. At the time the criminal case was filed on Dec 18, 1974, the law in force vesting jurisdiction upon said court is the Judiciary Act of 1948. General Order No. 49 dated Oct 4, 1974, redefines the jurisdiction of military tribunals over certain offenses, and estafa and malversation are not enumerated therein. Therefore, the Military Commission is not vested with jurisdiction over the crime of estafa. We do not have here a situation involving two tribunals with concurrent jurisdiction over a particular crime so as to apply the rule that whoever takes cognizance first acquires jurisdiction exclusive of the other. The Military Commission is without power or authority to hear and determine the crime of estafa against Mariano hence there is no concurrent jurisdiction to speak of. Estafa falls within the sole exclusive jurisdiction of civil courts. UNITED STATES VS. JUEVES The defendants/appellants were charged with the crime of brigandage or highway robbery for having committed the following acts: (1) On the evening of December 31, 1903, in Alabat, Tayabas Province, a band of armed men entered the municipal building, bound the presidente, took seven guns, and killed the Justice of Peace of that town. (2) On the evening of February 04, 1904, a band of some twenty men ( 3 armed with guns and the rest with bolos ) entered the house of Doroteo Maraver, situated in the

35232938 Crimpro Digest

Embed Size (px)

DESCRIPTION

crim

Citation preview

Page 1: 35232938 Crimpro Digest

People vs. Mariano Facts: The office of the Provincial Fiscal of Bulacan filed an Information accusing Mariano of estafa. Mariano was the Liaison Officer of Mayor Nolasco and is authorized to receive and be receipted for US excess property of USAID/NEC for the use and benefit of the municipality. The property received were electric cables and cable powers amounting to P4,797.35 which he had a duty to deliver to the Mayor. However he willfully, unlawfully and feloniously, with grave abuse of confidence and deceit, misappropriate, misapply and convert to his own personal use and benefit the items. Mariano filed a motion to quash the Information claiming that the court had no jurisdiction. He claimed that the items which were the subject matter of the Information against him were the same items for which Mayor Nolasco was indicted by the Military Commission under a charge of malversation and found guilty. He claimed that inasmuch as the case against Mayor Nolasco has already been decided by the Military Tribunal, the CFI of Bulacan had lost jurisdiction over him. Respondent judge granted the motion to quash stating that since the Military Commission had already taken cognizance of the malversation case involving the same subject matter in its concurrent jurisdiction with the Court, the case for estafa has already been heard and decided. Issue: Whether or not civil courts and military commissions exercise concurrent jurisdiction over estafa and committed by a civilian Held: there is no concurrent jurisdiction Ratio: The question of jurisdiction of respondent CFI is to be resolved on the basis of the law or statute providing for or defining its jurisdiction. The Judiciary Act of 1948 in Section 44 (f) provides the CFI shall have original jurisdiction in all criminal cases in which the penalty provided by law is imprisonment for more than six months or fine of more than 200 pesos. Estafa falls under the original jurisdiction of CFI. Jurisdiction of a court is determined by the statute in force at the time of the commencement of the action. At the time the criminal case was filed on Dec 18, 1974, the law in force vesting jurisdiction upon said court is the Judiciary Act of 1948. General Order No. 49 dated Oct 4, 1974, redefines the jurisdiction of military tribunals over certain offenses, and estafa and malversation are not enumerated therein. Therefore, the Military Commission is not vested with jurisdiction over the crime of estafa. We do not have here a situation involving two tribunals with concurrent jurisdiction over a particular crime so as to apply the rule that whoever takes cognizance first acquires jurisdiction exclusive of the other. The Military Commission is without power or authority to hear and determine the crime of estafa against Mariano hence there is no concurrent jurisdiction to speak of. Estafa falls within the sole exclusive jurisdiction of civil courts. UNITED STATES VS. JUEVES The defendants/appellants were charged with the crime of brigandage or highway robbery for

having committed the following acts:

(1) On the evening of December 31, 1903, in Alabat, Tayabas Province, a band of armed men entered the municipal building, bound the presidente, took seven guns, and killed the Justice of Peace of that town. (2) On the evening of February 04, 1904, a band of some twenty men ( 3 armed with guns and the rest with bolos ) entered the house of Doroteo Maraver, situated in the

Page 2: 35232938 Crimpro Digest

sitio of Capalohan (at that time, a part of the town of Capalongan, Ambos Camarines ), tied and bound four men, and ordered the women to prepare a meal for them. After eating, they left with them men whom they bound and also some tobacco and rice. They went to the house of Francisco Ambas, one of their prisoners, where they took a hog and other things. Afterwards, they proceeded to a river where they liberated Angelo Lunasco and Doroteo Maraver ( two of their prisoners ). (3) On Holy Thursday of 1904, a band of men, armed with a revolver, 3 shotguns, and bolos, went to the barrio of Basiad ( also within the jurisdiction of Capalongan, Ambos Camarines at that time ) taking 3 men as prisoners, conducting them from place to place. One of the prisoners, Juan Talento, managed to escape. (4) One morning of August 1910, two of the accused, Agustin Jueves and Felix Jueves, with their younger brother, Esteban Jueves, all armed with large bolos, entered the house of Serapio Juego, situated near the sea of sitio of Pangas, municipality of Calauag, Tayabas, and took possession of a small quantity of rice.

Graciana Laiman, wife of Francisco Ambos, and Doroteo Maraver identified all seven of the

accused of having committed the unlawful acts. Angelo Lunasco identified 5 of the accused and Juan Talento identified 3 of the accused of

having committed the unlawful acts. The testimonies of the witnesses show conclusively that the guilt of the appellants has been

established beyond any question of doubt. NOTE: A small portion of the Province of Ambos Camarines was transferred to the Province of

Tayabas. The Court of First Instance of Tayabas took jurisdiction of the crime committed within the

transferred territory PRIOR to the time jurisdiction was conferred and convicted the appellants.

ISSUES:

1.) W/N the Court of First Instance of Tayabas has jurisdiction of the crime committed within the transferred territory PRIOR to the time jurisdiction was conferred. 2.) W/N the assumption of jurisdiction over crimes committed before jurisdiction was conferred is in violation of the ex post facto clause of the Philippine Bill.

RATIO:

1.) YES. The territory where the acts complained of in the case at bar were committed having been transferred to the Province of Tayabas prior to the institution of this action, the court of that province had jurisdiction to hear and determine this case.The change of the territory after the crime was committed and before the institution of this action does not touch the offense nor change the punishment therefor. It only includes the place of the commission of the offense within another judicial district, and subjects the Appellants to trial in that district. The court held that the jurisdiction of Tayabas court was complete. 2.) NO. The assumption of jurisdiction over crimes committed before jurisdiction was conferred is not in violation of the ex post facto clause of the Philippine Bill having said that the change of the territory after the crime was committed and before the institution of this action does not touch the offense nor change the punishment. It does not prejudice the rights of the accused.

DECISION: The Supreme Court held that CFI of Tayabas has jurisdiction of the crime and affirmed the conviction of the appellants. Reyes v. Diaz Facts: Some election protest case (it didn‘t expound)

Page 3: 35232938 Crimpro Digest

Doctrine:

1) The issue of jurisdiction which confers appellate powers upon this Court in a given case is not such question as is dependent exclusively upon minor matters of fact or upon a mere construction of the pleadings, but that which has reference to the more important question of jurisdiction of the trial court over the subject-matter as determined by law.

2) Jurisdiction over the subject-matter is the power to hear and determine cases of the general class to which the proceedings in question belong and is conferred by the sovereign authority which organizes the court and defines its powers. The question of whether a court has jurisdiction over the subject-matter, calls for interpretation and application of the law of jurisdiction which distributes the judicial power among the different courts in the Philippines, and since the ruling on the matter is of far-reaching consequences, affecting, as it may, the very life and structure of our judicial system, the law has deemed it wise to place the power and authority to act thereon in the highest court of the land.

3) In order that a court may validly try and decide a case, it must have jurisdiction over the persons of the parties. But in some instances it is said that the court should also have jurisdiction over the issue meaning thereby that the issue being tried and decided by the court be within the issues raised in the pleadings. But this kind of jurisdiction should be distinguished from jurisdiction over the subject-matter the latter being conferred by law and the former by the pleadings. Jurisdiction over the issue, unlike jurisdiction over the subject-matter, may be conferred by consent either express or implied of the parties. Although an issue is not duly pleaded it may validly be tried and decided if no timely objection is made thereto by the parties. This cannot be done when jurisdiction over the subject-matter is involved. In truth, jurisdiction over the issue is an expression of a principle that is involved in jurisdiction over the persons of the parties. Where, for instance, an issue is not duly pleaded in the complaint, the defendant cannot be said to have been served with process as to that issue. At any rate, whether or not the court has jurisdiction over a specific issue is a question that requires nothing except an examination of the pleadings, and this function is without such importance as call for the intervention of this Court.

Velunta vs. Chief, Philippine Constabulary Facts:

Petitioner is a regular member of the Integrated National Police (INC) of Tacloban City, with rank of Patrolman.

On April 16, 1982, at around 6pm while directing traffic, petitioner tried to apprehend Romeo Lozano, a tricycle driver, for violation of traffic rules and regulations. An altercation occurred between them and resulted in the shooting at the left cheek leading to the death of Romeo Lozano.

The widow of Lozano, Mrs. Lozano, filed an administrative complaint against petitioner in NAPOLCOM.

NAPOLCOM issued a decision which held petitioner guilty of grave misconduct with a penalty of Dismissal from Service.

On a motion for reconsideration, petitioner was held guilty for Less Grave Misconduct and modified the penalty to suspension for 6 months without pay.

During the pendency of the administrative case, Mrs. Lozano filed a complaint for homicide with the City Fiscal‘s Office of Tacloban.

After finding prima facie evidence of petitioner‘s intent to kill and he was on the performance of his duties when the shooting occurred, the City Fiscal recommended the case be referred to the Tanodbayan for further investigation.

The Tanodbayan endorsed the filing of information for homicide against petitioner.

The case was referred to the military authorities pursuant to P.D.1850 which authorizes the Chief of Phil. Constabulary (PC) to convene court martials to try, hear, and decide cases for criminal acts committed by members of the INC.

Page 4: 35232938 Crimpro Digest

Petitioner challenges the assumption of jurisdiction by the General Court Martial over his case pursuant to E.O 1040 in relation to E.O. 1012 whereby supervision and control over all units and members of the INC have been placed under NAPOLCOM directly under the Office of the President.

Relevant Issue:

1. Whether the General Court Martial has jurisdiction over the case. Held:

1. Yes

Ratio: 1. It is specifically stated under EO 1012 that only the “operational supervision and

direction” over all units of the INC force stationed or assigned in the diff. cities and municipalities that was transferred from the PC to the city or municipal govt. concerned. Under EO 1040, it is the exercise of “administrative control and supervision” over all units of the INC forces throughout the country that was transferred to the President of the Phils. The distinction between operational supervision and direction over the INC and jurisdiction or authority of a court-martial to hear, try and decide a criminal proceeding is that in the former, it refers to how the police will perform their functions and who shall direct such performance, while in the latter, it refers to the tribunals vested with power to try criminal cases against them.

Subido v. Sandiganbayan (G.R. No. 122641; January 20, 1997) Facts: On June 25, 1992, Bayani Subido Jr., then a Commissioner of the Bureau of Immigration and Deportation (BID) and Rene Parina, a BID special agent, while in the performance of their official functions, issued and implemented a warrant of arrest against James J. Maksimuk, knowing fully well that the BID decision requiring Maksimuk‘s deportation was not yet final and executory. This resulted to the detention of Maksimuk for a period of 43 days, causing him undue injury. Subido and Parina were charged with Arbitrary Detention defined and punished by Article 124 of the Revised Penal Code. For their part, the petitioners filed a Motion to Quash, contending that the Sandiganbayan had no jurisdiction over the case since when it was filed, Subido was no longer part of the service and Parina was not occupying a position corresponding to salary grade ―27‖. Issue: Whether or not the Sandiganbayan had jurisdiction over the case Ruling: Yes. The Sandiganbayan had jurisdiction over the case by virtue of Section 2 of R.A. 7975, which amended Section 4 of P.D. No. 1606

Section 2: Section 4 of P.D. No. 1606 is hereby further amended to read as follows:

Section 4: Jurisdiction – The Sandiganbayan shall exercise original jurisdiction in all cases involving: a. Violations of Republic Act No. 3019, as amended, otherwise known as the Anti-Graft and Corrupt Practices Act, Republic Act No. 1379, and Chapter II, Section 2, Title VII of the Revised Penal Code, where one or more of the

Page 5: 35232938 Crimpro Digest

principal accused are officials occupying the following positions in the government, whether in permanent, acting or interim capacity, at the time of the commission of the offense;

1) Officials of the executive branch occupying positions of regional director and higher, otherwise classified as grade 27 and higher, of the Compensation and Position Classification Act of 1989 (R.A. 6758), specificially including:

xxx 5) All other national and local officials classified as Grade ―27‖ and

higher under the Compensation and Position Classification Act of 1989.

b. Other offenses or felonies committed by the public officials and employees mentioned in subsection (a) of this section in relation to the office. c. Civil and criminal cases filed pursuant to and in connection with the Executive Order Nos. 1,2, 14, and 14-A. In cases where none of the principal accused are occupying positions corresponding to salary grade ―27‖ or higher, as prescribed in said R.A. 6758, or PNP officers occupying the rank of superintendent or higher, or their equivalent, exclusive jurisdiction thereof shall be vested n the proper Regional Trial Court, Metropolitan Trial Court, Municipal Trial Court, and Municipal Circuit Trial Court, as the case may be, pursuant to their respective jurisdiction as provided in Batas Blg. 129.

Contrary to the claims of the petitioners, R.A. 7975 applies since what is considered is the time of the commission of the crime, during which Subido was still Commissioner of BID. Similarly, although Parina was holding a position with a classification lower than salary grade ―27‖, it still applies to him since he is prosecuted as a co-conspirator of Subido, the principal accused. Jurisdiction is only vested on the other courts if none of the principal accused where occupying positions corresponding to salary grade ―27‖. CUYCO VS. SANDIGANBAYAN G.R. Nos. 137017-18, February 8, 2000 FACTS:

Graft Investigation Officer Ma. Lourdes M. Vilaria-Yap found probable cause for the indictment of Ramon G. Cuyco (petitioner), Generoso P. Germino and Melcy V. Wee for violation of Section 3(a), R.A.3019, and Cuyco with Rolando R. Madarang for violation of Section 3(e) of the same Act.

She recommended the filing of two informations against Cuyco.

The Ombudsman approved the recommendation, and the prosecution filed with the Sandiganbayan two informations against Cuyco for both offenses.

Cuyco filed a motion to quash the information for lack of jurisdiction, contending that the Sandiganbayan had no jurisdiction over the cases.

The Sandiganbayan issued resolutions denying the motion to quash and ordering the preventive suspension of Cuyco and his co-accused for ninety (90) days.

Cuyco filed a motion for reconsideration seeking to set aside the resolutions in question and to dismiss the criminal cases for want of jurisdiction. This was denied.

Page 6: 35232938 Crimpro Digest

ISSUE:

Whether or not the Sandiganbayan had jurisdiction over the cases against petitioner for violation of Sections 3(a) and (e), R.A.3019, as amended.

HELD:

No. The Sandiganbayan did not have jurisdiction.

RATIO:

The Sandiganbayan has jurisdiction over offenses and felonies, whether simple or complexed with other crimes committed by public officers and employees in relation to their office, where the accused holds a position with salary grade "27" and higher under the Compensation and Position Classification Act of 1989.

At the time of the commission of the offense in 1992, he was occupying the position of Director II, Salary Grade 26, hence, jurisdiction over the cases falls with the Regional Trial Court.

The Sandiganbayan has no jurisdiction over violations of Section 3(a) and (e), R.A., as amended, unless committed by public officials and employees occupying positions of regional director and higher with Salary Grade "27" or higher, under the Compensation and Position Classification Act of 1989 (R.A. 6758) in relation to their office.

Buaya v. Polo Facts:

Buaya is the insurance agent of Country Bankers Insurance Company. She is required to make a periodic report and accounting of her transactions and remit premium collections to Country Bankers. The account of Buaya was audited and it was showed that P358,850 was unremitted.

She was charged with estafa in the Manila RTC. Buaya filed a motion to Quash on the ground that the Manila RTC has no jurisdiction because she is from Cebu and that the case is civil in nature (a separate civil case was filed) but was denied. She filed for a motion for reconsideration but it was also denied. So now, she filed a certiorari in the Supreme Court questioning the denials of such motion by Judge Polo.

The case did not go through the Court of Appeals Issue: Did the court have jurisdiction over the case? Held: YES. Manila RTC has jurisdiction over the case.

Estafa was sufficiently addressed in the information. Estafa is a continuing or transitory offense and the case can be instituted upon any place where any of the essential requisites of Estafa were committed. One of the essential requisites of Estafa is damage or prejudice to 3

rd persons. Here, given that the main office of Country Bankers is in Manila, the non

remittance of Buaya which caused damage to Country Bankers was committed in Manila.

Page 7: 35232938 Crimpro Digest

The information in this case reads as follows: “that during the period 1980 to June 15, 1982, inclusive, in the City of Manila, Philippines, the accused did then and there willfully, unlawfully and feloniously defraud the Country Bankers Insurance Corp represented by Banez duly organized and existing under the Philippine laws of the Philippines, with principal address at 9

th floor, GR Antonio Bldg, TM Kalaw, ERmita in said city in the following manner:

the said accused…. “ The City of manila was alleged in the information so the Manila RTC has jurisdiction.

US v Gallegos Facts:

Criminal complaint for adultery

Mariano Gallegos had relations with Benita Antioquia several times, and as a result Antioquia gave birth to a daughter

Complaint includes both Gallegos and Antioquia, but only Gallegos was arrested, because Antioquia “could not be apprehended after due diligence”

Issue: 1. W/N, in a criminal action for adultery one of the defendants may be tried alone when for some

reason or other his co-defendant has not been arrested and brought to trial 2. W/N prosecution, after closing its case, may present additional proof relating to the jurisdiction of the

court Held/Ratio: 1. Yes. While the law provides that the complaint must be presented, in a criminal action for adultery (a)

by the offended person and (b) against both of the alleged culprits, there is no provision of law requiring that they shall be tried jointly. In fact, there is a positive provision of law permitting them to be tried separately (section 33, general orders no. 58)

2. Yes. Section 31, General Orders no. 58 provides that after both parties have presented their evidence, they may offer rebutting testimony only, unless the court, in the furtherance of justice, permit them to offer new and additional evidence bearing upon the main issue in question.

The question of jurisdiction of the court is always a question of importance. If evidence is necessary to prove that fact, so far as the place of the commission of the rime is concerned, in the interest of justice the court may always permit it to present additional evidence, if that fact appears before the trial of the case is closed

REPUBLIC vs. SUNGA June 20, 1988 FACTS:

An information for Attempted Homicide was filed by the Provincial Fiscal of Camarines Sur against accused-private respondents Rafael Anadilla, Ariston Anadilla and Jose Anadilla. A hearing was set but was postponed since Rafael Anadilla was not yet arrested by the authorities. The court a quo issued an order for the arrest of said accused, and at the same time set a new trial date.

However, 4 months before the trial date, the court a quo issued the now assailed order which reads:

Page 8: 35232938 Crimpro Digest

“Considering that the offended party, Jose Dadis is no longer interested in the further prosecution of this case and there being no objection on the part of the accused Ariston Anadilla, Rafael Anadilla and Jose Anadilla, this case is hereby DISMISSED with costs de oficio.

Consequently, the order of arrest issued by this Court against the accused Rafael Anadilla dated March 11, 1974, is hereby ordered lifted and has no force and effect.

The bail bond posted for the provisional liberty of the accused is hereby ordered cancelled.

In the case of Ariston Anadilla and Jose Anadilla, the Provincial Warden is hereby ordered to release said accused from their detention immediately upon receipt of this order.

SO ORDERED. “

The order was based on an AFFIDAVIT OF DESISTANCE which was executed and notarized by the victim and mentioned that:

a. he was no longer interested in the further prosecution of the case b. he had forgiven the accused c. his material witnesses could not be located, and that without their testimonies, the

guilt of the accused could not be proven beyond reasonable doubt.

The provincial fiscal moved for reconsideration of the dismissal, but was also denied. Hence the petition and issue of the case.

ISSUE:

Whether or not the court a quo may dismiss a criminal case on the basis of an affidavit of desistance executed by the offended party, but without a motion to dismiss filed by the prosecuting fiscal.

RATIO:

The court cites a similar case Crespo v. Mogul in its when it answered that the filing of a complaint or information in Court initiates a criminal action. The Court thereby acquires jurisdiction over the case, which is the authority to hear and determine the case. When after the filing of the complaint or information a warrant for the arrest of the accused is issued by the trial court and the accused either voluntarily submitted himself to the Court or was duly arrested, the Court thereby acquired jurisdiction over the person of the accused.

The preliminary investigation conducted by the fiscal for the purpose of determining whether a prima facie case exists warranting the prosecution of the accused is terminated upon the filing of the information in the proper court.

The rule is that once a complaint is filed, the disposition of the accused rests in the sound discretion of the court. The fiscal cannot impose his opinion on the court when the case has been submitted to it as his jurisdiction ends in the direction and control of the prosecution of the case. Only the court can decide what the best direction is for the case, as it is within its exclusive jurisdiction.

In this case, almost 10 years have elapsed since the date of the filing of the information, hence it was not unusual that the victim could not find his witnesses, the testimonies of whom are needed to convict the accused. The fiscal still believed that he could convict the accused without thee testimonies in his MR!

Page 9: 35232938 Crimpro Digest

Although the Crespo doctrine holds that it is the courts duty to judge whether a case should be dismissed, any move of the offended part to dismiss the case, even without objection of the accused, should first be submitted to the fiscal. It is only after the fiscal’s hearing that the court should exercise its duty to continue or dismiss the case.

Petition dismissed.

US v De la Santa *This case was decided in 1907, when the old Penal and Civil Codes were still in effect. The old Penal Code provision on seduction covered the age range of 12-23, while the old Civil Code places a woman‘s age of majority at 23. Facts:

De la Santa allegedly seduced Teofila Sevilla in 1902, under promise of marriage, when she was under 21.

The complaint was filed only in 1906, when she was already 24.

It was at Esteban Sevilla’s instance, private prosecutor and Teofila‘s dad, that the complaint was filed.

The Marinduque CFI convicted De la Santa of Seduction.

Hindi na dumaan sa CA yung case. Issues: 1. Does the father have such right to institute a Seduction complaint, taking Teofila‘s age (of

majority, upon filing) into consideration? 2. Does De la Santa‘s lack of objection over the complaint, being outside the CFI‘s jurisdiction,

effectively make him waive his right to such defect? 3. With Teofila‘s attendance and testimony in the CFI proceedings, may she be said to have

instituted the proceedings, though the complaint was signed and sworn by her father? Held/Ratio 1. No. Article 448 of the old Penal Code

i does not automatically authorize the institution of

criminal proceedings by all the mentioned parties. Though the act may only be committed upon a woman still legally incapacitated, all other alternative parties mentioned, like the father, lose their right to do so when the victim reaches the age of majority.

Though mentioned disjunctively, the right to institute criminal proceedings in seduction is exclusively and successively conferred in the order they are named. (victim muna, then parents, then grandparents, then guardian)

Also, the lawmakers did not intend this conjunction of rights, bearing in mind the possibility of pardon by the offended party while these other persons are attempting to file complaints.

2. No. ―Lack of jurisdiction over the subject matter is fatal, and subject to objection at any stage of the proceedings,‖ in the trial court or on appeal.

Jurisdiction may only be conferred by law, and the accused cannot confer jurisdiction, by waiver or otherwise.

3. No. Nothing on record indicated such, and the fact that she was subpoenaed compelled her to appear, voluntary or not. (beside the point daw, taking active part in the proceedings is not sufficient basis for this unless she herself submitted and formally maintained the complaint) Note

1 Criminal proceedings for seduction can only be instituted on the complaint of the

offended person or her parents, grandparents, or guardian. x x x If the person injured should, by reason of her age or mental condition, lack the requisite

personality to appear in court, and should, besides, be wholly unprotected, not having parents, grandparents, brothers, or guardian of person or property to denounce the crime, the procurador sindico or the or the public prosecutor may do so, acting on the strength of public rumor.

Page 10: 35232938 Crimpro Digest

In all the cases of this article the express or implied pardon of the offended party shall extinguish penal action or the penalty, if it should have been already imposed on the culprit.

The pardon shall never be presumed, except by the marriage of the offended party with the offender.

US vs Reyes Facts The defendant was charged with the crime of estafa and falsification. He is an employee of the Manila-Dagupan Railway. He issued a ticket to a passenger who was going from Manila to Caloocan who continued his trip to Malolos. The ticket issued that simulated the trip was from Manila to Bocaue which costs 18 cents. There was a 1 peso and 22 cents difference in the fare. The complaint does not designate the place where the falsification was committed. Issue: Where must the jurisdiction be vested? The case must be tried in Tarlac and not in Manila.

According to the testimony of the accused, he rendered an account to the station master

at Tarlac of the Money collected during the trip. It is also there where he delivered the

stub in which the simulation or falsification was committed.

In an itemized account of the collections made him on the trip, an entry corresponding to

the stub alleged to have been falsified was dated in Tarlac and contains an invoice of

delivery signed by the accused and a receipt signed by the station master for the sum of

6 pesos and 48 cents.

Because it was within the territory of the court that the appropriation constituting the

crime of estafa was committed and the accused made use of the document alleged to be

false at the same territory.

PEOPLE OF THE PHILIPPINES V ALEX REGALARIO ET.AL. GR NO. 101451 MARCH 23,1993

Facts: Regalario and 6 others were found guilty of murder of a Menardo Garcia. (CrimPro relevant) Judgment of conviction was promulgated on Jan 17 and a copy of which was received by the appellants‘ counsel the next day, Jan 18. Appellants filed MR on Jan 31 but the court denied on Feb 22. On March 14, appellants filed notice of appeal but the trial court denied for having been filed out of time. (Syllabus topic relevant)Neither the State nor private prosecutor moved for the dismissal of the appeal made by the appellants nor objected to the order of the trial court to forward the records to the SC for appellate review Issue:

1. On the matter of filing within the reglamentary period:

Whether the lower court erred in denying notice of appeal on the ground of being filed out

of time

Appellants‘ claim: computation of 15 days within which to file notice of appeal should

have been counted from Feb 23(day after the MR was denied) and not from Jan 31

(day the verdict pre-MR was given)

o In short, the appellants‘ claim that the 15 day reglamentary period should

have restarted on the day of the denial of the MR

2. On the matter concerning the syllabus topic, Estoppel by laches to bar attacks on

jurisdiction

(Counsel for the state questions authority of the Supreme Court to review the case)

Ruling of the lower court: (Issue on procedure is being questioned for the first time in the Supreme Court) Ruling of the Supreme Court: (TRIAL COURT DENIAL OF NOTICE OF APPEAL AFFIRMED):

1. The notice of appeal was filed beyond the reglamentary period set by law

Page 11: 35232938 Crimpro Digest

Section 6 of Rule 122 states that: ―…appeal must be taken 15 days from

promulgation or notice of judgment or order appealed from. This period for perfecting

an appeal shall be interrupted from the time a motion for new trial or

reconsideration is filed until notice of order overruling the motion shall have been

served upon the accused or his attorney...‖

o The rule states period shall only be ―interrupted‖ thus appellants only had 1

day with which to file notice of appeal with the trial court

2. Estoppel by laches to bar attacks of jurisdiction of the court had already attached

Neither the public or private prosecutor moved for the dismissal of the appeal or objected

to the order of the trial court to forward records to the Supreme Court for appellate

review. It was only after appellants had already filed their briefs with the SC that counsels

for the state raised the issue of belated appeal and lack of appellate jurisdiction of the SC

in the case.

Dela Cruz vs. Moya Facts: -Dela Cruz is a Member of the Armed Forces Intelligence and Operations Section - armed with a MISSION ORDER, DelaCruz proceeds to Maco, Davao del Norte to investigate reports of illegal cockfighting being conducted - Delacruz and company catches operators of cockfighting, including deceased Eusebio Cabito in flagrante -Delacruz and co.fails to arrest operators but confiscates evidence of the crime (eg. Gaffs, fighting cocks, etc) -Delacruz and co. were followed by the cockfight operators on their way back to the PC headquarters, fighting ensued wherein Delacruz shot Cabito Aug 2, 1979 Delacruz is charged with homicide in the CFI of Davao Issues: W/N CFI has jurisdiction over the subject matter Held: NO. Ratio: -Civil Procedure; Jurisdiction; One of the essential requisites of a valid court proceeding is that the court hearing must have jurisdiction over of the subject matter of the case. Determined by the statute at force at the time the action was commenced -at that time General Order.59 was operative giving military tribunals exclusive jurisdiction over all offenses committed by military personnel while in the performance of their official duty -Delacruz was executing a Mission Order=performing official duties -court records contain a copy of Mission Order; certificate from secretary of DND is unnecessary -CFI was without jurisdiction to try the case People vs Magallanes Facts:

Two informations for kidnapping for ransom with murder (of Gargar and Lumangyao)

were filed with the RTC of Bacolod City against 14 persons, 5 whom are members of the

PNP

Each of the accused pleaded not guilty upon arraignment

Prosecution rested its case and the trial court started to receive the evidence for the

accused

Private prosecutors moved for the transmittal of the records of the cases to the

Sandiganbayan (SB) on the ground that, pursuant to our decision of 11 March 1994

in Republic of the Philippines vs. Asuncion, the RTC has no jurisdiction over the cases

Page 12: 35232938 Crimpro Digest

because the offenses charged were committed in relation to the office of the accused

PNP officers

RTC issued an order denying the motion

The prosecution, represented by the OSG, prayed for a TRO challenging the refusal of

the respondent Judge Magallanes to transfer the cases to the SB

Issue: w/n RTC of Bacolod City has jurisdiction over the case instead of the SB - YES Ruling:

At the time the informations in the said cases were filed, the law governing the jurisdiction

of the SB was Section 4 of P.D. No. 1606

Jurisdiction is determined by the allegations in the complaint or information, and not by

the result of evidence after trial.

For lack of an allegation in the informations that the offenses were committed in relation

to the office of the accused PNP officers or were intimately connected with the discharge

of the functions of the accused, the subject cases come within the jurisdiction of the RTC

and not of the SB

The allegation of "taking advantage of his position" or "taking advantage of their

respective positions" incorporated in the informations is not sufficient to bring the

offenses within the definition of "offenses committed in relation to public office."

The SB partly lost its exclusive original jurisdiction in cases involving violations of R.A.

No. 3019 (Anti-Graft and Corruption), it retains only cases where the accused are those

enumerated in subsection a, Section 4 (R.A. 7975) and officials classified as Grade "27"

and higher under the Compensation and Position Classification Act of 1989 (R.A. No.

6758)

Also, upon express provision of Section 7 of R.A. No. 7975, all criminal cases in which

trial has not yet begun in the SB shall be referred to the proper courts. Hence, cases

which were previously cognizable by the SB under P.D. No. 1606, as amended, but are

already under the jurisdiction of the courts by virtue of the amendment introduced by R.A.

No. 7975, shall be referred to the latter courts if hearing thereon has not yet been

commenced in the SB

CONCURRING and DISSENTING OPINION:

- Padilla: the act of Torres was undoubtly connected with his position as station

commander of PNP; others were performing as law enforcers such that the case

must be prosecuted in the SB. The informations must have been filed with the SB.

- Kapunan: in favor of granting bail

Uy v. CA Facts:

Uy formed a partnership with Leong to contribute to the former‘s lumber business.

A lumber store was erected using the funds Leong contributed.

Their relationship soured and this prompted Leong to ask for her investment back.

The checks issued by Uy were all dishonored for insufficiency of funds.

Leong lodged a complaint for Estafa and a violation of BP 22.

Manila RTC acquitted Uy from Estafa but was convicted for BP 22.

Issue:

Whether or not the RTC of Manila acquired jurisdiction over the violations of the

Bouncing Checks Law

Page 13: 35232938 Crimpro Digest

Whether the doctrine of jurisdiction by estoppel applies

Held:

The RTC of Manila did not acquire jurisdiction.

No.

Ratio:

It is a fundamental rule that for jurisdiction to be acquired by the courts in criminal cases,

the offense should have been committed or anyone of its essential ingredients took place

within the territorial jurisdiction of the court. The various dealings were held in Manila,

and thus the RTC of Manila acquired jurisdiction over the Estafa case. However, there is

no evidence that shows that jurisdiction over the BP 22 was acquired. (related sa issue

ng crimpro)

The Revised Rules on Criminal Procedure, under Rule 117 Sec.3, states that the

accused may move to quash the complaint or information on the ground of (b) the court

trying the case has no jurisdiction over the accused or the offense charged. Sec. 8 of the

same rule says that the failure of the accused to assert any ground of a motion to quash

before he pleads to the complaint or information, either because he did not file a motion

to quash or failed to allege the same in said motion, is deemed a waiver of the grounds of

a motion to quash. However, Section 3(b) states that an exception to this is the ground

of lack of jurisdiction over the offense charged. In this case, it cannot be said that Uy is

estopped to question the jurisdiction of the RTC of Manila even if she questioned it 5

years into the trial already. It can be questioned at any stage of the proceeding.

Binay vs. Sandiganbayan FACTS:

Office of the Ombudsman filed before the Sandiganbayan 3 separate informations against Binay for violation of Art 220 RPC, 2 violations of Sec3(e), RA3019, alleging that these crimes were committed during Binay‘s incumbency in Makati

Sandiganbayan denied motion to quash (filing of charges to filing of information took 6 years thus denied his right to due process

Sandigan granted motion to suspend accused pendente lite and ordered him suspended for 90 days

RA 7975 took effect redefining the jurisdiction of the sandigan

Binay filed a motion to refer his case to the ―proper court‖ for further proceedings, alleging that the resolutions denying motion to quash and granting preventive suspension were issued when the Sandiganbayan had already lost jurisdiction over the subject cases (DENIED)

Municipal Mayor-classified as grade 27; at the time of the commission of the offenses, Binay was municipal mayor, although he received salary less than that received by grade 27, the compensation act was not yet in existence at the time of the commission of the crime…At the time the act was passed, Binay was receiving salary under grade 28

SC issued a TRO based on Binay‘s motion

Magsaysay-mayor of San Pascual, Batangas; charged w/ violation of RA3019, w/ the RTC of Batangas; another info. was filed w/ the Sandiganbayan, for overpricing a landscaping project

Magsaysay filed a motion to quash the info. w/ the Sandiganbayan-no jurisdiction over the matter-DENIED

prosecutor Mallonga filed a petition w/ the RTC to refer the case to the Sandiganbayan by virtue of RA7975

SC: it is the Sandiganbayan w/c has jurisdiction over the cases

Page 14: 35232938 Crimpro Digest

*when RA7975 took effect, Binay had not yet been arraigned in the Sandiganbayan; on the other hand, the RA was already in effect when the info. against Magsaysay was filed w/ the RTC; while the cases were pending, RA8249 was enacted ISSUE:

1. Had the sandigan been ousted of its jurisdiction over the case of municipal mayor after the pages of RA 7975 coupled with the filing earlier of an information for the same offense before the RTC having territorial jurisdiction and venue of the commission of the offense

-Binay and Magsaysay contend that they do not come under the exclusive original jurisdiction of the Sandiganbayan because 1)at the alleged time of the commission of the offenses, they were not classified as grade 27; 2)municipal mayors are not included in the enumeration of Sec.4(a)(1) of PD1606 as amended by RA7975; 3)congressional records reveal that the law did not intend municipal mayors to come under the exclusive original jurisdiction of the Sandiganbayan; also contend that pay scales determine salary grade classifications HELD: SANDIGANBAYAN RETAINS JURISDICTION OVER BINAY‘s CASE

the constitution states that in providing for the standardization of compensation of govt officials and EEs, congress shall take into account the nature of the responsibilities pertaining to, and the qualifications required for the positions;

RA6758 provides that differences in pay are to be based upon substantive differences in duties and responsibilities, and qualification requirements of the positions; grade, as defined in PD958: all classes of positions w/c, although different with kind of subject matter or work, are sufficiently equivalent as to level of difficulty and responsibilities and level of qualification requirements of the work to warrant the inclusion of such classes of positions w/in one range of basic compensation; grade w/c determines salary; possible that an official‘s salary may be less than prescribed for his grade…

to determine WON the Sandiganbayan has jurisdiction, reference should be made to RA6758 and the index of occupational services, position titles and salary grades; salary grade is a matter of law, not of proof; index lists municipal mayors under salary grade 27, therefore, are under the exclusive original jurisdiction of the Sandiganbayan

…the rule is that where a court has already obtained and is exercising jurisdiction over a controversy, its jurisdiction to proceed to the final determination of the cause is not affected by new legislation placing jurisdiction over such proceedings in another tribunal. The exception to the rule is where the statute expressly provides, or is construed to the effect that it is intended to operate as to actions pending before its enactment. Where a statute changing the jurisdiction of a court has no retroactive effect, it cannot be applied to a case that was pending prior to the enactment of the statute.

RA7975, Sec.7, belongs to the exception rather than the rule; the provision is transitory in nature and expresses the legislature’s intention to apply its provisions on jurisdiction to criminal cases in w/c trial has not begun in the Sandiganbayan

the trial of Binay had not yet begun as of the date of the approval of RA7975, therefore, the Sandiganbayan retains jurisdiction over the case

Binay invokes the rule that the jurisdiction of a court once it attaches cannot be ousted by subsequent happenings or events, although of such character w/c would have prevented jurisdiction from attaching in the 1

st instance; filing of the info. in Sandiganbayan-

subsequent happening or event no application; RTC never had jurisdiction over the case

Page 15: 35232938 Crimpro Digest

estoppel cannot be successfully invoked; jurisdiction is determined by law, not by the consent or agreement of the parties or by estoppel; estoppel remains the exception rather than the rule

no double jeopardy when the accused enters a plea in a court that has no jurisdiction

G.R. No. 120011 September 7, 1999

LT. COL. LINO A. SANCHEZ and MAJOR VICENTE S. MANAGAY, petitioners, vs. THE SANDIGANBAYAN, THE OMBUDSMAN and THE PEOPLE OF THE PHILIPPINES, respondents.

Facts: Petitioners are officers of the Philippine Army (PA). Lt. Col. Lino A. Sanchez at times material hereto was Commanding Officer, 9th Post Engineer Detachment, Headquarters and Headquarters Support Group (HHSG), Philippine Army. Major Vicente S. Managay was G-4, HHSG, Philippine Army.

A pre-trial investigating officer, submitted a report to the Commanding General, Philippine Army, stating that there was a prima facie case against petitioners for violation of the Articles of War for causing the wrongful release of P599,547.00 for payment of renovation of an office in the Phil. Army.

On the basis of the report, they initiated court martial proceedings against petitioners before the Philippine Army Permanent General Court Martial. Both of the petitioners were arraigned before the Court Martial and both pleaded not guilty.

The investigating officers also referred the findings to the Provincial Prosecutor while there were court martial proceedings against the petitioners recommending the filing of an information with the Sandiganbayan against petitioners for violation of R.A. No. 3019. The Provincial Prosecutor of Rizal endorsed the records to the Ombudsman. The Ombudsman filed with Sandiganbayan information against them for the violating the said RA. The petitioners filed a motion to dismiss the case for lack of jurisdiction and was denied through a resolution stating that the offenses charged in the court martial are distinct and separate from each other. Thus, the petitioners were also arraigned in Sandiganbayan and they pleaded not guilty.

Petitioners filed a motion for reconsideration of the denial reiterating that the Sandiganbayan had no jurisdiction over the case as the court martial had acquired original and exclusive jurisdiction over the case, pursuant to R.A. No. 7055, and that the acts complained of in the charge sheet in the court martial and the Information before the Sandiganbayan were the same or identical. The Sandiganbayan denied the motion for lack of merit.

Hence, the petitioners filed the case to the Supreme Court.

Issue:

W/N Sandiganbayan had jurisdiction over the case.

Ruling:

Page 16: 35232938 Crimpro Digest

No. Though the Sandiganbayan had jurisdiction when the case was filed, it no longer had jurisdiction over the case under RA 7975. This law removing the jurisdiction of Sandiganbayan over the case was passed before the motion for reconsideration was submitted. Therefore, the Sandiganbayan acted without jurisdiction when the motion for reconsideration was denied.

LACSON v. EXECUTIVE SECRETARY Adherence of Jurisdiction (Exceptions) & Action of the Court when determined that it has no jurisdiction & Jurisdiction of the SB (Offense deemed committed in relation to Public Office) Facts:

11 persons believed to be members of the Kuratong Baleleng gang, an organized crime syndicate involved in bank robberies, were slain by elements of the Anti-Bank Robbery and Intelligence Task Group (ABRITG). Petitioner Lacson and petitioners-intervenors Acop and Zubia were members of ABRITG.

SPO2 de los Reyes exposed to the media that what actually happened between the members of the Kuratong Baleleng and the ABRITG was a summary execution (rub-out) and not a shoot-out. Ombudsman Desierto formed a panel to investigate the incident. Upon investigation, all the PNP officers and personnel allegedly involved in the incident were absolved from any criminal liability because it was a legitimate police operation. However, a review board modified the panel‘s ruling and recommended the indictment for multiple murder against 26 respondents, including Lacson, Acop and Zubia. The Ombudsman approved the recommendation and 11 informations for murder were filed against Lacson, as principal, Acop and Zubia as accessories before the Sandiganbayan‘s Second Divisio

Upon motion by all the accused in the 11 informations, the SB allowed them to file a motion for reconsideration of the Ombudsman‘s action. After a reinvestigation, the Ombudsman filed 11 amended informations before the SB, wherein Lacson was charged only as an accessory.

The accused filed separate motions questioning the jurisdiction of the SB, asserting that under the amended informations, the cases fall within the jurisdiction of the RTC pursuant to Section 2 of R.A. 7975. They said that the said law limited the jurisdiction of the SB to cases where one or more of the ―principal accused‖ are government officials with Salary Grade 27 or higher, or PNP officials with the rank of Chief Superintendent or higher. They did not qualify under the said requisites because the highest ranking principal has the rank of only a Chief Inspector and none has the equivalent of at least SG 27. Thus, the SB admitted the amended information and ordered the cases transferred to the QC RTC, which has original and exclusive jurisdiction under R.A. 7975.

The Office of the Special Prosecutor moved for a reconsideration, insisting the cases should remain with the SB. Petitioner and some of the accused opposed.

Pending the motions for reconsideration, RA 8249 was approved amending the jurisdiction of the SB by deleting the word ―principal‖ from the phrase ―principal accused‖ in Section 2 (a & c) of R.A. 7975. Even before the issue of jurisdiction came up with the filing of the amended informations, the house bill for that was already introduced in Congress. ISSUES:

(1) WON Sections 4 & 7 of R.A. 8249 is unconstitutional.

a. WON the statute violates the petitioners‘ right to due process and equal

protection clause because the provisions seemed to have been designed for the

Sandiganbayan to continue to acquire jurisdiction over the case.

b. WON the statute is an ex-post facto law.

(2) Whether the case falls within the Sandiganbayan’s or Regional Trial Court’s

jurisdiction.

a. WON the offense of multiply murder was committed in relation to the office

of the accused PNP officers, making the case fall within the

Sandiganbayan’s jurisdiction.

HELD: (1) SECTIONS 4 AND 7 OF R.A. 8249 ARE CONSTITUTIONAL.

Page 17: 35232938 Crimpro Digest

The issue on due process and equal protection is too shallow to deseve merit. There were no concrete evidence and convincing argument presented. The classification made by the law was reasonable and not arbitrary. There is nothing ex post facto in the statute. Generally, ex post facto laws deal with the retroactive effect of penal laws and the said R.A. is procedural in nature. (2) THE REGIONAL TRIAL COURT HAS EXCLUSIVE ORIGINAL JURISDICTION OVER

THE CASES.

For a case to be within the jurisdiction of the Sandiganbayan, it must be shown that the offense charged in the information was committed in relation to the office of the accused.

In People v. Montejo, the court held that an offense is said to have been committed in relation to the office if it is “intimately connected” with the office of the offender and perpetrated while he was in the performance of his official functions. This “intimacy” must be alleged in the information, which is what determines the jurisdiction of the court. What is controlling is the specific factual allegations in the information that would show the close intimacy between the discharge of the accused’s official duties and the commission of the offense charged. It does not even matter whether the phrase “committed in relation to his office” appears in the information or not.

In the case at bar, what the amended information contains is a mere allegation that the offense was committed by the accused public officer in relation to his office and that is not sufficient. Such phrase is merely a conclusion of law. Since it was not proven that the crime of murder was committed in the discharge of their duties, the Sandiganbayan does not have jurisdiction over the cases. US v Morales Facts: Morales was convicted of brigandage under Section 1 Act No. 518 by the CFI in Manila. He contended, however, that CFI Manila has no jurisdiction on him because he was arrested in Malabon and was subsequently brought to Manila from their place of arrest contrary to Sec.1 Act No 518 which states that person guilty of brigandage must be punished in the place in which they may be taken or from which they may have fled. Issue: Does CFI Manila have jurisdiction? Held and Ruling: No, it does not have jurisdiction since Morales was arrested in Malabon and not in Manila. Pursuant to Sec. 1 Act No. 518, they must be punished in the place where they may be taken or where they may have fled. REPUBLIC VS. ASUNCION FACTS: Private Respondent Alexander Dionisio Y Manio, a member of the Philippine National Police (PNP) assigned to the Central Police District Command Station 2 in Novaliches, Quezon City, was dispatched by his Commanding Officer to Dumalay Street, Novaliches to respond to a complaint that a person was creating trouble there. Dionisio proceeded there, where he subsequently shot to death T/Sgt. Romeo Sadang. Pursuant to Sec. 7, Rule 112 of the Rules of Court, the Office of the City Prosecutor filed a case with the Regional Trial Court (RTC) of Quezon City an information charging Dioniso with the crime of homicide. While trial was already in progress, the respondent Judge issued an order, motu propio, an order requiring the prosecution and the defense to comment on whether the Court should still proceed with the trial of the case. “In view of the decision of the SC in the case of Deloso vs. Domingo, the Sandiganbayn has jurisdiction over offenses committed by public officers when the penalty prescribed by law for the offense id higher than prision correcional. The murder charge against petitioner carries the penalty of reclusion temporal in its maximum period to death hence it is cognizable by the Sandiganbayan, and the Ombudsman has primary jurisdiction to investigate it.”

Page 18: 35232938 Crimpro Digest

Respondent Judge dismissed the Criminal Case ―for re-filing with the Sandiganbayan‖ on the ground that the Sandiganbayan, and not the RTC has jurisdiction over the case. Private prosecutor moved for reconsideration, citing the opinion of the Secretary of Justice that ―crimes committed by PNP members are not cognizable by the Sandiganbayan‖ because ―they fall within the exclusive jurisdiction of the regular courts‖ as provided in Section 46 of R.A. No. 6975 and ―the Sandiganbayan is not a regular court but a special court.‖ The respondent judge denied the motion. Hence this petition ISSUE:

Whether or not R.A. NO. 6975 VESTS THE EXCLUSIVE JURISDICTION IN CRIMINAL CASES INVOLVING PNP MEMEBERS ONLY IN THE ―REGULAR COURTS‖ WHICH EXCLUDES THE SANDIGANBAYAN SINCE IT IS CONSTITUTIONALLY AND STATUTORILY A ―SPECIAL COURT‖.

RATIO:

INTERPRETATION OF THE TERM “REGULAR COURTS” IN R.A. 6975 Police forces have traditionally been under the civil authority. The overwhelming sentiment of the framers of the 1987 Constitution against martial law regime and the militarization of the police forces prompted them to explicitly direct the establishment and maintenance of one police force. R.A No. 6975 on the PNP are intended to implement Section 6, Article XVI of the 1987 Constitution, ―national in scope and civilian in character. This civilian character is unqualified and unconditional and is therefore, all-embracing. The Declaration of Policy (Section 2) of R.A. No. 6975 faithfully carried out this mandate. The civilian character refers to its orientation and structure. The mandate of Section 46 R.A 6975 is to divest courts-martial of any jurisdiction over criminal cases involving PNP members and to return or transfer that jurisdiction to the civil courts and was explicitly provided for in the original Section 68 of the House Bill No. 23614. The terms civil courts and regular courts were used interchangeably or were considered as synonymous by the Bicameral Conference Committee and then by the Senate and House of Representatives. Accordingly, the term regular courts in Section 46 of R.A. No. 6975 means civil courts. There could have been no other meaning intended since the primary purpose of the law is to remove from courts-martial the jurisdiction over criminal cases involving members of the PNP.

IS SANDIGANBAYAN INCLUDED IN THAT TERM

Regular Courts are those within the judicial department of the government, namely, the Supreme Court and such lower courts as may be established by law. Per Section 16, Chapter 4, Book II of the Administrative Code of 1987, such lower courts ―include the Court of Appeals, Sandiganbayan, Court of Tax Appeals, Regional Trial Courts, Shar‘a District Courts, Metropolitan Trial Courts, Municipal Trial Court, Municipal Circuit Trial Courts, and Shari‘a Circuit Courts. The Sandiganbayan was created by P.D. No. 1486 pursuant to the mandate of Section 5, Article xiii of the 1973 Constitution. This was revised by P.D. 1606 and amended by P.D. 1860 then 1861. Under the amendments introduced by P.D. No. 1861, the Sandiganbayan has jurisdiction over the following cases: ―a. Exclusive original jurisdiction: Violations of Anti-Graft and Corrupt Practices Act, R.A. 1379, and Chapter 2 Title VII of the RPC; 2. Other offenses or felonies committed by public officers and employees in relation to their office…b. Exclusive appellate jurisdiction ‖ Undoubtedly then, Sandiganbayan is a regular court and is thus included in the term regular courts of Sec. 46 R.A. No. 6975. Courts of special jurisdiction, which are permanent in character, are also regular courts. The Sandiganbayan is a court with special jurisdiction because its creation as a permanent anti-graft court is constitutionally mandated and its jurisdiction is limited to certain classes of offenses This is further strongly indicated by Sec. 1 of P.D. No. 1606 which vests upon it ―all the inherent powers of a court of justice‖ and places it on ―the same level as the

Page 19: 35232938 Crimpro Digest

Court of Appeals,‖ and by Section 4 thereof, as amended by P.D. No. 1861, which grants it appellate jurisdiction over certain case decided by the RTC, MeTC, MTC, and MCTC.

***That the public officers or employees committed the crime in relation to their office must, however, be alleged in the information for the Sandiganbayan to have jurisdiction over a case under Sec. 4(a)(2). This allegation is necessary because of the unbending rule that jurisdiction is determined by the allegations of the information. WHEREFORE, judgment is hereby rendered ORDERING the respondent Judge to conduct a preliminary hearing to determine whether the crime charged was committed by the private respondent in relation to his office and

1. If it was in relation to his office, DIRECTING the respondent judge to transmit the records of the case to the Sandiganbayan which shall docket and proceed with the case as if the same were originally filed with it; or

2. If he determines otherwise, DIRECTING him to set aside the Challenged Orders, to proceed with the hearing of the case and to render judgment thereon.

MORALES V. CA GR No.: 126623 Date: December 12, 1997 Petitioner: Ernesto Morales Respondents: Court of Appeals, Hon. Alfredo Gustilo, as presiding judge of RTC, and People of the Philippines Facts Petitioner was charged for violating the Dangerous Drugs Act of 1972 in an information filed before the Regional Trial Court (RTC). He then filed a Motion to Dismiss on the ground the penalty for the offense charged should not exceed prision correccional or six years‘ worth of imprisonment and that it is the Metropolitan Trial Court that has jurisdiction over the case. In denying this motion, the RTC reasons out that, while MTC has exclusive jurisdiction over cases with penalties of not more than six years of imprisonment, an exception is provided in the said Act. It provides the Court of First Instance (currently, the RTC) shall have ―concurrent original jurisdiction over all offenses punishable under the Act.‖ Petitioner then filed a petition for certiorari before the Court of Appeals (CA). CA dismissed the petition for lack of jurisdiction over the case. Hence, this petition In raising the same to SC, petitioner furthers that, since only about 0.5 gram of shabu was involved, the imposable penalty would not exceed prision correccional. Therefore, RTC doesn‘t have jurisdiction over his case. Issue Whether RTC has jurisdiction to try petitioner‘s alleged violation of the Dangerous Drugs Act Held

Page 20: 35232938 Crimpro Digest

WHEREFORE, the petition is GRANTED, but only insofar as the issue of jurisdiction of respondent Court of Appeals in CA-G.R. SP No. 40670 is concerned. The Resolutions of 8 August and 13 September 1996 of the Court of Appeals are SET ASIDE, while the challenged orders in Criminal Case No. 96-8443 of the Regional Trial Court of Pasay City, Branch 116, are AFFIRMED. The trial court is hereby DIRECTED to proceed with the trial of Criminal Case No. 96-8443 with all reasonable dispatch. Ratio

Whether RTC has jurisdiction to try petitioner‘s alleged violations of the Dangerous Drugs Act

YES. Section 32 of RA 7691 reads:

Sec. 32. Jurisdiction of Metropolitan Trial Courts, Municipal Trial Courts and Municipal Circuit Trial Courts in Criminal Cases. — Except in cases falling within the exclusive original jurisdiction of Regional Trial Court and of the Sandiganbayan, the Metropolitan Trial Courts, Municipal Trial Courts, and Municipal Circuit Trial Courts shall exercise:

(2) Exclusive original jurisdiction over all offenses punishable with imprisonment not exceeding six (6) years irrespective of the amount of fine, and regardless of other imposable accessory or other penalties, including the civil liability arising from such offender or predicated thereon, irrespective of kind, nature, value or amount thereof: Provided, however, That in offenses involving damage to property through criminal negligence, they shall have exclusive original jurisdiction thereof.

It must be noted, however, that the exclusive jurisdiction of these courts does not cover those cases which, by law, fall within the RTC‘s and Sandiganbayan‘s exclusive jurisdiction, regardless of the prescribed penalty. In the case at bar, the Dangerous Drugs Act specifically confers upon the RTC the jurisdiction over cases such as this. Sanchez vs. Demetriou Facts: The Presidential Anti Crime Commission requested the filing of charges against Sanchez in connection with the rape-slay of Mary Eileen Sarmenta and Allan Gomez. DOJ conducted a preliminary investigation. A PNP Commander ‗invited‘ Sanchez for questioning. He was identified by two people who executed extrajudicial confessions implicating Sanchez as principal in the rape-slay. He was put on arrest status and taken to DOJ. A warrant was subsequently issued from the RTC of Manila. Cases were filed in the RTC of Laguna but the case was transferred to the RTC of Manila. Motion to quash was filed because 1) Only the Ombudsman had the competence to investigate and 2) As a public officer, he can only be tried by the Sandiganbayan. Issues: 1) Did the Ombudsman have the sole competence to investigate? 2) Can he be tried in the regular courts? Held and Ratio: 1) No. Though the Ombudsman is empowered to investigate and prosecute illegal acts of public officials, the authority is not an exclusive authority but rather a shared or concurrent authority

Page 21: 35232938 Crimpro Digest

2) Yes. There is no direct relation between the commission of the crime of rape with homicide and the petitioner‘s office as municipal mayor because public office is not an essential element of the crime charged. The offense can stand independently of the office. (The case did not state the decisions of the lower courts.) Bartolome v People Facts: Rolando Bartolome and Elino Coronel are both officers of Ministry of Labor, NCR, Manila. They conspired to falsify the Civil Service Personal Data Sheet of Bartolome making it appear that he passed the career service exam and that he was a 4

th year AB student of FEU. Ricardo

Buenviaje, Tanodbayan Special Prosecutor filed a case at the Sandiganbayan accusing them of the crime of Falsification of Official Document. Sandiganbayan convicted them. Petitioners now question the validity of Sandigabayan‘s ruling. Issue: Whether or not Sandiganbayan has jurisdiction over petitioners‘ case SC Ruling: Proceedings in Sandiganbayan are null and void ab initio. Under Section 4 of PD 160, Sandiganbayan shall have jurisdiction over:

1. Violations of Anti-Graft and Corrupt Practices and RA 1379

2. Crimes committed by public officers and employees, including those in (Gov‘t Owned or

Controlled Corporations) GOCCs, embraced in Title VII of the RPC

3. Other crimes or offenses committed by public officers or employees including those

employed in GOCCS in relation to their office.

Nowhere does the act of falsification belong to the abovementioned cases where the Sandiganbayan have jurisdiction. Falsification of Public Document is embraced in Title IV of the RPC. Perhaps the nearest reason for Sandiganbayan‘s taking cognizance of the case is scenario 3 in the above enumeration. However, it does not appear that the official positions of Bartolome and Coronel were connected to the offense they committed. They did not need to be government employees to falsify a public document. In the light of the Montilla case, it was upheld that when public office is not an essential ingredient of the offense such that the offense can‘t exist without the office, it does not fall under Sandiganbayan‘s jurisdiction.

Natividad v. Felix February 4, 1994

Facts: PNP requested Tarlac Provincial Prosecutor to investigate Mayor Natividad‘s involvement in the death of Lourdes Aquino‘s husband, Severino Aquino, at the Ramos Police Station. During the investigation, PNP filed another complaint with the Tarlac Municipal Circuit Court, which directed Mayor Natividad‘s arrest with bail. He posted bail with the Manila RTC, which later issued an order recalling warrant. After conducting the preliminary investigation, MCTC determined that there was probable cause to hold Natividad for murder with bail. The Provincial Prosecutor approved the filing of information against Natividad and Llerina in the Tarlac RTC (where Hon. Felix was the judge). Warrant of arrest was issued. Upon seeing that the MCTC judge failed to conduct the second staged in the preliminary investigation, RTC recalled the warrant and remanded the case for further preliminary investigation. A panel of prosecutors later held that probable cause exists. The information was amended, additionally charging Cabaong, Llerina and Millado. An arrest warrant was issued without bail.

Page 22: 35232938 Crimpro Digest

Mayor Nativadad alleged that there was no preliminary investigation, and that Hon. Felix had no jurisdiction because it was the Ombudmans, not the provincial prosecutor, who had jurisdiction to conduct the preliminary investigation and that the proper court was the Sandiganbayan. Hon. Felix denied Mayor Natividad‘s motion, and committed the latter to Tarlac Penal Colony. Issue: Whether or not Hon. Felex committed grave abuse off discretion in admitting the amended information filed by the provincial fiscal and in directing Natividad‘s arrest Held: No. Ratio: The latest law on Sandiganbayan (PD 1606) states that there are 2 requirements for an offense to fall under the Sandiganbayan‘s jurisdiction: 1) offense committed by a public officer must be in relation to his office, and 2) that the penalty be higher that prision correccional or imprisonment for 6 years or a fine of P6,000. The 2

nd requirement was met, but the 1

st requirement wasn‘t because the offense charged wass

murder. The offense could not have been committed in the performance of the mayor‘s responsibility to maintain peace and order. The alleged act doesn‘t fall under any of the functions of the municipal mayor in the Local Government Code. Assuming arguendo that the 1

st requirement was satisfied, Ombudsman has only primary

jurisdiction over cases cognizable by the Sandiganbayan, not exclusive original jurisdiction. The Ombudsman is not an exclusive authority but a concurrent authority with similarly authorized agencies. He may take over the investigation at any stage from any investigative government agency. His investigatory powers are but directory in nature. (Also, Natividad wasn‘t denied due process as he has been afforded every opportunity to present his counter-affidavit. He was notified, but it was he who did not appear.) Olaguer v Regional Trial Court Facts: Philippine Journalists Inc (PJI) executed a mortgage in favor of Developmen Bank of Philippines(DBP) for certain financing accomodations. PJI assigned 67% of stocks to DBP. DBP appointed certain PJI stockholders as proxies. PJI failed to comply with its obligation to DBP leading to the cancellation of appointment of petitioners and designated Olaguer, Velez, and De Leon (Petitioners). Olaguer also asked some of respondent to assign shares not only to the three proxies by DBP but also to two others to be chosen by him so they can sit in the PJI board of directors. Though Olaguer was voted chairman of the board and CEO of PJI, he failed to comply with his commitment which gave respondents to cancel the assignment. He also did some illegal acts which gave rise to several complaints in court against herein Petitioners. Before the cases were resolved, then president Cory Aquino terminated his his appointment as member of the board of directors of DBP. Despite the termination, Olaguer still continued with the performance of his functions. There was an agreement enetered into b DBP and herein respondents calling for a special stockholders meeting to elect a new board of directors. Olaguer contends that the agreement cannot be implemented because Olaguer claims that he has just been designated the fiscal and team leader of the PCGG assigned to PJI and that all his actions are sanctioned and reported to PCGG, and that it is PCGG which exercises the voting rights of all PJI common stocks sequestered since 1986. .

Page 23: 35232938 Crimpro Digest

Olaguer then filed a motion to dismiss the cases against him on the ground that the court has no jurisdiction over the persons of herein petitioners. The motion to dismiss was denied thus this petition Issue W/N the trial court has jurisdiction over the case Held No, the trial court has no jurisdiction over the case. It is the Sandiganbayan who has jurisdiction over PCGG cases. Ratio There is no dispute that PJI is now under sequestration by the PCGG and that civil case no. 0035 was filed in Sandiganbayan wherenin the PJI is listed as among the corporations involved in the unexplained wealth wealth case against Marcos. Records also show that Olaguer was acting in behalf of the PCGG and under Section 2 of EO !$, the Sangiganbyana has exclusive and original jurisdiction over all cases regarding the unexplained wealth of Marcos. The decision of the Sandiganbayan is subject to review on certiorari exclusively by the Supreme Court. In the exercise of its functions, the PCGG is a co-equal body with the RTC and co-equal bodies have no power to control the other.

PRESIDENTIAL COMMISSION ON GOOD GOVERNANCE V PEÑA

159 SCRA 556

FACTS:

PCGG issued an order freezing the assets, effects, documents and records of two export garment

manufacturing firms, American Inter-Fashion Corporation and De Soleil Apparel Manufacturing

Corporation. Both firms have been organized by joint venture agreement with the approval of the Textile

Export Board. 2/3 of the two corporations were subscribed to Local Investors and the 1/3 was subscribed to

Hongkong Investors.

The PCGG appointed Saludo as OIC of the two corporations with authority to manage and operate it.

Saludo and Yeung Chun Ho were the authorized signatories to effect deposits and withdrawals of the funds

of the two corporations. Yam Kam Shing became a co-signatory of Yeung Chun Ho. However, in a

Memorandum addressed to the depository banks Saludo revoked the authorization previously issued

because Yiam Kam Shing was a Hongkong Chinese International staying in the country on a mere tourist

visa. When OIC Saludo withdrew money from Metropolitan Bank and Trust Company for the payment of

salaries Yim Kam Shing instituted an action for damages with a writ of preliminary injunction against

Metropolitan bank, PCGG, PCGG Commissioner Bautista and OIC Saludo. He questions the revocation of

the authorization as signatory previously granted to Yim Kam Shing as a co-representative.

PCGG filed a motion to dismiss on the ground that the Regional Trial court has no jurisdiction over PCGG

or over the subject of the case.

The Judge denied PCGG’s motion to dismiss.

The PCGG filed this petition to the SC questioning the jurisdiction of the Trial Court over it.

ISSUES:

I. Whether Regional Trial Courts have jurisdiction over the PCGG and properties sequestered and placed in

its custodia legis in the exercise of its powers.

II. Whether Regional Trial Courts may interfere with and restrain or set aside the orders and actions of

PCGG.

HELD: I. NO. The RTC do not have jurisdiction over the PCGG.

II. NO. Jurisdiction over all sequestered cases of ill-gotten wealth, assets and properties under the past

regime fall within the exclusive and original jurisdiction of the Sandiganbayan, subject to review only by

the SC.

Page 24: 35232938 Crimpro Digest

RATIO:

I. The RTC and the CA have no jurisdiction over the PCGG in the exercise of its powers under applicable

Executive Orders and Article XVIII, section 26 of the Constitution. It may not interfere with and restrain or

set aside the orders and actions of PCGG.

Under Section 2 of EO 14, all cases of the PCGG regarding the “Funds, moneys, assets, and properties

illegally acquired or misappropriated by Former Pres. Marcos, Mrs. Imelda Marcos, their close relatives,

subordinates, business associates, dummies, agents, or nominees” whether civil or criminal, are lodged

within the exclusive and original jurisdiction of the Sandiganbayan and all incidents arising from, incidental

to, or related to, such cases necessary fall likewise under the Sandiganbayan’s exclusive and original

jurisdiction, subject to review on certiorari exclusively by the Supreme Court.

II. In the exercise of quasi-judicial functions, PCGG is a co-equal body with the Regional Trial Court and

co-equal bodies have no power to control the other.

EO no 14, section 2 also provides that the PCGG “shall file all such cases, whether civil or criminal, with the

Sandiganbayan which shall have exclusive and original jurisdiction thereof”. All those who wish to question

or challenge the PCGG’s acts or orders must seek recourse in the Sandiganbayan, which is vested with the

exclusive and original jurisdiction. The Sandiganbayan’s decisions and final orders are in turn subject to

review on certiorari exclusively by the SC.

Olaguer v. Military Commission No. 34, G.R. No. L-54558, 22 May 1987, 150 SCRA 144. FACTS: Petitioners were detained by the military for subversion. The Armed Forces Chief of Staff created Military Commission No. 34 to try the petitioners‘ criminal case. Petitioners filed for a petition for habeas corpus with the Supreme Court. While the petition was pending, the Military Commission convicted and sentenced the petitioners to death by electrocution. ISSUE: Do military commissions or tribunals have the jurisdiction to try civilians for crimes allegedly committed during martial law when civilian courts are open and functioning? RULING: The trial contemplated in the due process clause of the Constitution is by judicial and not by executive or military process. Military commissions or tribunals are not courts within the Philippine judicial system. By the principle of separation of powers, interpreting the law as to when a person has violated it is a judicial function vested in the judiciary. As long as civil courts are open and functioning, military courts have no jurisdiction over civilian offenses. Also, Presidential Proclamation No. 2045 lifted martial law and revoked General Order No. 8 (creating military tribunals). Cases under military courts should be immediately transferred to civilian courts. Abadilla vs Ramos

FACTS:

A group of officers and enlisted men of the AFP seized control of the radio-television broadcasting facilities of the Republic Broadcasting System (GMA-Channel 7). The takeover might have been a prelude to similar operations throughout the national capital, however it did not succeed. After the incident another group of enlisted men staged a mutiny inside the Fort

Page 25: 35232938 Crimpro Digest

Bonifacio military facility in Makati, Metropolitan Manila. The mutiny, dubbed as "The Black Saturday Revolt," did not succeed either.

The investigations for both incident disclosed that Colonel Rolando N. Abadilla of the Philippine Constabulary (PC) of the AFP was one of the leaders of both the unsuccessful takeover of the GMA radio-television facilities as well as ―The Black Saturday Revolt‖. The Board of Officers investigating the matter recommended that the case of Colonel Abadilla be endorsed for pre-trial investigation and that the appropriate charges be filed against him. A charge sheet was prepared against the Colonel.

Colonel Abadilla was at large when both investigations were conducted. Respondent Major General Renato De Villa, Commanding General of the PC and Vice-Chief of Staff of the AFP issued an Order for the arrest and confinement of Colonel Abadilla. Also, respondent AFP Chief of Staff General Fidel V. Ramos issued General Orders No. 342 dropping Colonel Abadilla from the rolls of regular officers of the AFP.

Later, the Assistant City Fiscal of Quezon City filed charges against Colonel Abadilla. Colonel Abadilla was arrested by a group composed of the Philippine Army and the PC. He was detained by the military. The spouse of Colonel Abadilla together with their minor children went to the Supreme Court and filed a petition for habeas corpus, challenging the validity of the detention of Colonel Abadilla. There main contention was (as to jurisdiction) when Colonel Abadilla was dropped from the rolls of officers, he became a civilian and as such, the order for his arrest and confinement is null and void because he was no longer subject to military law. On the other hand, the Solicitor General contends that military jurisdiction had fully attached on Colonel Abadilla inasmuch as proceedings were initiated against him before the termination of his service in the military.

ISSUE: Is the detention of Colonel Abadilla illegal? (The resolution of this issue will relate to the jurisdiction of the military authorities over the person of Colonel Abadilla.)

HELD

Col. Abadilla is under military jurisdiction

Ratio

As early as March, 1987, months before Colonel Abadilla was dropped from the rolls of officers, the military authorities began the institution of proceedings against him. As of that time, he was certainly subject to military law. He was under investigation for his alleged participation in the unsuccessful mutinies when he was an officer of the AFP. His superiors could not confine him during the period of investigation because he was at large. This disregard for military duty and responsibility may have prompted his superiors to cause him to be dropped from the rolls of officers. The fact that Colonel Abadilla was dropped from the rolls of officers cannot and should not lead to the conclusion that he is now beyond the jurisdiction of the military authorities. If such a conclusion were to prevail, his very own refusal to clear his name and protect his honor before his superior officers in the manner prescribed for and expected from a ranking military officer would be his shield against prosecution in the first place.

The military authorities had jurisdiction over the person of Colonel Abadilla at the time of the alleged offenses. This jurisdiction having been vested in the military authorities, it is retained up to the end of the proceedings against Colonel Abadilla. Well-settled is the rule that jurisdiction once acquired is not lost upon the instance of the parties but continues until the case is terminated. The rule that jurisdiction over a person is acquired by his arrest applies only to criminal proceedings instituted before the regular courts. It does not apply to proceedings under military law. At the time the military investigations were commenced, Colonel Abadilla was an officer of the AFP subject to military law. As such, the military authorities had jurisdiction over his person.

Page 26: 35232938 Crimpro Digest

His earlier arrest could not be effected because he was at large.

PS

The petitioner cited various cases with regards to the lost of jurisdiction of military tribunals over discharged from the service but the Court found that this cases were inapplicable to the case (one of the cases deals with being honorably discharged).

WILLIAM TAN, ET. AL. V. BARRIOS, ET. AL. G.R. Nos. 85481-82. October 18, 1990

Facts

Then President Ferdinand E. Marcos authorized the creation of military tribunals to try and decide cases of military personnel. Its jurisdiction was modified (exclusive of the civil courts) to include violations of the law on firearms and other crimes directly related to the quelling of rebellion and the preservation of the country‘s safety and security. Yet, the President may, in the public interest, refer to a Military Tribunal a case falling under the exclusive jurisdiction of the civil courts and vice versa.

Twelve people were arrested and charged for the crimes of murder through the use of an unlicensed or illegally possessed firearm and unlawful possession of a pistol with ammunition before the Military Tribunal. Since this was a celebrated case, Marcos withdrew his earlier order to transfer the case to the civil courts and for the Military Tribunal to retain it. All the accused pleaded not guilty. Martial rule ended and the military tribunals and commissions were abolished.

In Cruz vs. Enrile, SC nullified the proceedings in military courts against the civilians and ordered the re-filing of informations against them in the proper civil courts. In Olaguer vs. Military Commission No. 34, et al. (Olaguer was sentenced to death without receiving evidence in his defense), SC said that even during the period of martial law as long as those courts are open and functioning, military commissions and tribunals have no jurisdiction over civilians charged with criminal offenses properly cognizable by civil courts. It declared unconstitutional the creation of the military commissions to try civilians and nullified the proceedings convicting non-political detainees.

Without conducting an investigation, State Prosecutor Barrios filed with RTC two informations for the mentioned crimes. He also failed to provide the RTC Judge the supporting affidavits and the SC order which is the basis of filing them because there is no SC order to re-file the criminal cases. The petitioners filed before SC a petition for certiorari to annul the informations. RTC issued warrants of arrest. SC issued a TRO and ordered the respondents to comment.

The Solicitor General argued that the proceedings involving civilians before a military commission were null and void because SC ruled in Olaguer that military tribunals are bereft of jurisdiction over civilians, hence, their decisions, whether of conviction or acquittal; do not bar re-prosecution for the same crime before a civil court. Barrios defended the re-prosecution on the ground that it will not constitute double jeopardy because the nullity of the jurisdiction of the military tribunal that acquitted them prevented the first jeopardy from attaching, thereby nullifying their acquittal.

Issue Whether or not Barrios committed grave abuse of discretion by re-filing the informations Held: YES. Ratio

The petitioners had been acquitted by the military court. Cruz vs. Enrile should be given a limited application for those who sought the annulment of the court martial proceedings and prayed for retrial in the civil courts, not all who had been tried and sentenced by a court martial during the period of martial law. The petitioners‘ re-prosecution, based on Cruz in which they took no part and were not heard, would be violative of their right to due process. In Cruz, the Secretary of Justice was ordered to file the necessary informations in the proper civil courts but it did not

Page 27: 35232938 Crimpro Digest

nullify the court martial proceedings against the other civilians petitioners who: (1) had finished serving their sentences; (2) had been granted amnesty; or (3) had been acquitted by the military courts. SC applied retroactively the rule on re-prosecution before the proper civil courts and prospectively for those released from custody.

The doctrine in Olaguer is that the trial of civilians by military process was not due process. However, only in particular cases where the convicted person or the State shows that there was serious denial of the Constitutional rights of the accused should the nullity of the sentence be declared and a retrial be ordered. If a retrial, is no longer possible, the accused should be released since the judgment against him is null on account of the violation of his constitutional rights and denial of due process.

The doctrine of "operative facts" applies to the case. The principle of absolute invalidity of the jurisdiction of the military courts over civilians should not be allowed to obliterate the "operative facts" if, such as in this case, the proceedings were fair, there were no serious violations of their constitutional right to due process, and that the jurisdiction of the military commission that heard and decided the charges against them during the period of martial law, had been affirmed by this Court.

The re-filing of the information against the petitioners would place them in double jeopardy. Furthermore, depriving them of the protection of the judgment of acquittal rendered by the military commission would amount to an ex post facto law or ruling.

ADDITIONAL CASES Isip vs. People

He who alleges must prove his allegations. Isip is charged with the crime of estafa. Apparently, Isip received jewelries that he was supposed to sell. On the due date, he failed to return neither the jewelries nor the proceeds of the supposed sale to the rightful owner. In his petition, he claimed that the trial court in Cavite, the tribunal that tried the case, had no jurisdiction. He claimed that that the transaction was entered into in Manila. The Court rejected such claim stating that Isip failed to prove such allegations. In such case, jurisdiction of a court over the criminal case is determined by the allegations in the complaint or information. And once it is so shown the court may validly take cognizance of the case. However, if the evidence adduced during the trial shows that the offense was committed somewhere else, the court should dismiss the action for want of jurisdiction. LAND BANK OF THE PHILIPPINES VS. RENE RALLA BELISTA G.R. NO. 164631 FACTS Belista inherited 8 parcels of lot located in Ligao, Albay. Department of Agrarian Reform (DAR) placed the lots under the coverage of CARP for which she is claiming for just compensation. DAR and LBP estimated the lots at P277, 582.58 and P317, 259.31, respectively. Believing that her lots were grossly, she filed a Petition for Valuation and Payment of Just Compensation against LBP at the DARAB-Regional Adjudicator for Region V. The Adjudicator decided that subject lots are valued at P2.8M and ordered LBP to pay Belista. Both parties filed for motion for reconsideration. This time, the Adjudicator, reduced the value of the lots to P2.5M. Aggrieved, LBP, petitioned with the RTC. RTC Ruling: RTC dismissed the case motu proprio for failure to exhaust first administrative remedies and in violation of 2003 DARAB Rules of Procedure which states that appeal of Regional Adjudicator‘s decision should first be filed at DARAB and not the RTC. CA affirmed the ruling To summarize, LBP did this:

Adjudicator RTC CA RTC & CA said LBP should have done this:

Adjudicator DARAB RTC CA

Page 28: 35232938 Crimpro Digest

However, LBP contends that DARAB Rules of Procedure is not applicable to them because its effectivity began February 8, 2003 while petition was filed at RARAD on November 11, 2002. Furthermore ISSUE W/N CA was correct in ruling that decision of Adjudicator HELD 1. NO. Decision of DARAB may be appealed directly to the RTC. RATIO The dates are irrelevant to determine the jurisdiction of the RTC. SC ruled that, according to R.A. No. 6657, which instituted CARP, decision of the Adjudicator need NOT be appealed first to DARAB before a party can resort to the RTC sitting as Special Agrarian Court (SAC). Main ratio: Sec. 57 of RA 6657 provides that RTC sitting as SAC has original and exclusive jurisdiction shall have original and exclusive jurisdiction over all petitions for the determination of just compensation to landowner. If 2003 DARAB rules are to be applied, the RTC would have appellate jurisdiction, which is contradictory to RA 6657. Sub-ratio: Another source of conflict is Sec. 50 of RA 6657, which provides that DAR shall have exclusive jurisdiction over all matters involving the implementation of agrarian reform. SC made a distinction between the SAC and DAR. DAR as an administrative agency cannot be granted the exclusive power to exercise eminent domain, which is a judicial function. DAR is vested with preliminary determination for just compensation but subject to challenge in courts.

Antonino v. Desierto GR No. 144492 December 18, 2008 FACTS On October 3, 1963, Pres. Diosdado Macapagal issued Presidential Proclamation No. 168:

do hereby withdraw from sale or settlement and reserve for recreational and health resort site purposes, under the administration of the municipality of General Santos, subject to private rights, if any there be, a certain parcel of land of the public domain situated in the said municipality and more particularly described as follows:

Mr-1160-D Municipal Reservation The Municipal Government of General Santos Magsaysay Park A parcel of land (as shown on plan Mr-1160-D) situated in the barrio of Dadiangas, Municipality of General Santos, province of Cotabato. Containing an area of 52,678 sq. m.

On the other hand, the property subject of PP No. 168 was thereafter subdivided into three lots, namely: Lot Y-1 with an area of 18,695 sq. m., Lot X containing 15,020 sq. m. and Lot Y-2 with 18,963 sq.m., or a total of 52,678 sq.m. which is still equivalent to the original area. Pres. Marcos then issued PP No. 2273 amending PP No. 168 declaring Lot Y-1 and Lot Y-2 open to disposition under the provisions of Public Land Act. Thus, leaving only Lot X reserved for recreational and health resort site purposes. The private respondents applied for Miscellaneous Sales Patent over portions of Lot X. Public respondent Jonillo as Deputy Land Management Inspector, then recommended for the approval of the survey authority requested by the private respondents for Lot X. Within the same day, the Survey Authority was issued to private respondents by public respondent CENR Officer Rivera. As a result, Lot X was subdivided into 16 lots.

Page 29: 35232938 Crimpro Digest

Respondent City Mayor Nuñez, assisted by respondent City Legal Officer Nalangan issued 1

st Indorsements addressed to CENRO, DENR for portions of Lot X applied by private

respondents. Thereon, public respondents Jonillo and City Assessor Dinopol, together with recommendation for approval from respondent Rivera, submitted an appraisal of the lots requested by private respondents. Subsequently, respondent Jonillo prepared a letter-report addressed to the Regional Executive Director of DENR for each of the sixteen applicants recommending for the private sale of the subject lots without public auction. Respondent CENR Officer Rivera, also issued recommendation letters for each of the sixteen applicants addressed to the DENR Officer for the approval of the appraisal of the subject lots and of the private sale. A notice of sale was issued by respondent Diaz also on the same date stating therein that the subject lots will be sold. Certificates of Title were issued by the Register of Deeds of Gen. Santos City to the private respondents. Based on the foregoing, petitioner filed a verified complaint-affidavit before the Ombudsman against the private respondents together with the public respondents for violation of paragraphs e, g, and j, Section 3 of RA No. 3019, as amended, and for malversation of public funds or property through falsification of public documents. OMBUDSMAN’S RULING

The Ombudsman also ruled that public respondents Diaz, Borinaga, Momongan and Cruzabra were found to have regularly performed their official functions. Accordingly, the charges against the respondents were dismissed. ISSUE

W/N THE OMBUDSMAN COMMITTED GRAVE ABUSE OF DISCRETION, AMOUNTING TO LACK OR IN EXCESS OF JURISDICTION IN THE EXERCISE OF HIS PROSECUTORY FUNCTIONS, BY DISMISSING THE CHARGES AGAINST THE RESPONDENTS DESPITE CLEAR AND CONVINCING EVIDENCE OF DIRECT PARTICIPATION AND INVOLVEMENT IN THE CONSPIRACY TO CHEAT AND DEFRAUD THE CITY GOVERNMENT OF GENERAL SANTOS CITY THROUGH THE ILLEGAL DISPOSITION OF LOT X OF THE MAGSAYSAY PARK IN VIOLATION OF LAW AND ITS CHARTER RULING The petition is DISMISSED. The alleged grave abuse of discretion imputed to the Ombudsman is found wanting in this case. Thus, this Court finds no reason to deviate from the general rule. Indeed, while the Ombudsman's discretion in determining the existence of probable cause is not absolute, nonetheless, petitioner must prove that such discretion was gravely abused in order to warrant the reversal of the Ombudsman's findings by this Court. In this respect, petitioner fails. RATIO/DOCTRINE Section 27 of R.A. No. 6770 (The Ombudsman Act of 1989)

22 provides:

SEC. 27. Effectivity and Finality of Decisions. – (1) All provisionary orders of the Office of the Ombudsman are immediately effective and executory.

A motion for reconsideration of any order, directive or decision of the Office of the Ombudsman must be filed within five (5) days after receipt of written notice and shall be entertained only on any of the following grounds:

(1) New evidence has been discovered which materially affects the order, directive or decision;

(2) Errors of law or irregularities have been committed prejudicial to the interest of the movant. The motion for reconsideration shall be resolved within three (3) days from filing: Provided, That only one motion for reconsideration shall be entertained.

Page 30: 35232938 Crimpro Digest

Petitioner failed to establish that her Motion for Reconsideration was indeed filed on time. In this case, petitioner did not proffer any explanation at all for the late filing of the motion for reconsideration. After the respondents made such allegation, petitioner did not bother to respond and meet the issue head-on. We find no justification why the Ombudsman entertained the motion for reconsideration, when, at the time of the filing of the motion for reconsideration the assailed Resolution was already final. Even only on the basis of this fatal procedural infirmity, the instant Petition ought to be dismissed.

Under Sections 12 and 13, Article XI of the 1987 Constitution, and pursuant to R.A. No. 6770, the Ombudsman has the power to investigate and prosecute any act or omission of a public officer or employee when such act or omission appears to be illegal, unjust, improper or inefficient.

23 Well-settled is the rule that this Court will not ordinarily interfere with the

Ombudsman's exercise of his investigatory and prosecutory powers without good and compelling reasons that indicate otherwise. The rule is based not only upon respect for the investigatory and prosecutory powers granted by the Constitution to the Office of the Ombudsman, but upon practicality as well. A contrary rule would encourage innumerable petitions seeking dismissal of investigatory proceedings conducted by the Ombudsman, which would grievously hamper the functions of the office and the courts, in much the same way that courts would be swamped by a deluge of cases if they have to review the exercise of discretion on the part of public prosecutors each time they decide to file an information or dismiss a complaint by a private complainant. This rule is not absolute. The aggrieved party may file a petition for certiorari under Rule 65 of the Rules of Court when the finding of the Ombudsman is tainted with grave abuse of discretion amounting to lack or excess of jurisdiction, as what the petitioner did in this case, consistent with our ruling in Collantes v. Marcelo,

25 where we laid down the following exceptions

to the rule: 1. When necessary to afford adequate protection to the constitutional rights of the accused; 2. When necessary for the orderly administration of justice or to avoid oppression or multiplicity of actions; 3. When there is a prejudicial question that is sub judice; 4. When the acts of the officer are without or in excess of authority; 5. Where the prosecution is under an invalid law, ordinance or regulation; 6. When double jeopardy is clearly apparent; 7. Where the court has no jurisdiction over the offense; 8. Where it is a case of persecution rather than prosecution; 9. Where the charges are manifestly false and motivated by the lust for vengeance; 10. When there is clearly no prima facie case against the accused and a motion to quash on that ground has been denied. Grave abuse of discretion exists where a power is exercised in an arbitrary, capricious, whimsical or despotic manner by reason of passion or personal hostility so patent and gross as to amount to evasion of positive duty or virtual refusal to perform a duty enjoined by, or in contemplation of law.

26

SHARON CASTRO vs. HON. MERLIN DELORIA

G.R. No. 163586 January 27, 2009

FACTS:

On May 31, 2000, Sharon Castro was charged by the Ombudsman before the Regional Trial Court with Malversation of Public funds. It was found that as the Revenue Officer I of the Bureau of Internal Revenue, she appropriated, took, misappropriated, embezzled and converted her collections and other accountabilities worth P556,681.53 to her own personal use and benefit. Then on August 31, 2001, Castro filed a Motion to Quash on the grounds of lack of jurisdiction and lack of authority of the Ombudsman to conduct the preliminary investigation and to file the Information. She was claiming that the case filed against her

Page 31: 35232938 Crimpro Digest

was cognizable by the RTC and may be investigated only by the public prosecutor since she was a public employee with a salary grade 27.

RTC’S DECISION: The RTC denied Castro‘s Motion to Quash since jurisdiction of the RTC over the case did not depend on the salary grade of petitioner, but on the penalty imposable upon the latter for the offense charged.

Moreover, it pointed out that the Supreme Court expressly

recognizes in the case of Uy vs. Sandiganbayan the prosecutorial and investigatory authority of the Ombudsman in cases is cognizable by the RTC.

COURT OF APPEALS’S DECISION: Affirmed the decision of the RTC ISSUE:

Whether the Ombudsman had the authority to file the same in light of this Supreme Court‘s ruling in the First "Uy vs. Sandiganbayan" case, which declared that the prosecutorial powers of the Ombudsman is limited to cases cognizable by the Sandiganbayan.

RATIO: The power to investigate and to prosecute granted by law to the Ombudsman is plenary and unqualified. It pertains to any act or omission of any public officer or employee when such act or omission appears to be illegal, unjust, improper or inefficient. Section 15 of RA 6770 gives the Ombudsman primary jurisdiction over cases cognizable by the Sandiganbayan. The law defines such primary jurisdiction as authorizing the Ombudsman "to take over, at any stage, from any investigatory agency of the government, the investigation of such cases." Said exercise of primary jurisdiction is not incompatible with the discharge of his duty to investigate and prosecute other offenses committed by public officers and employees. To carry out this duty, the law allows him to utilize the personnel of his office and/or designate any fiscal, state prosecutor or lawyer in the government service to act as special investigator or prosecutor to assist in the investigation and prosecution of certain cases. The law likewise allows him to direct the Special prosecutor to prosecute cases outside the Sandiganbayan‘s jurisdiction in accordance with Section 11(4c) of RA 6770. This goes to show that the Office of the Special Prosecutor is merely a component of the Office of the Ombudsman and may only act under the supervision and control and upon authority of the Ombudsman. Its power to conduct preliminary investigation and to prosecute is limited to criminal cases within the jurisdiction of the Sandiganbayan.

SUPREME COURT’S DECISION:

Supreme Court dismissed the case and held that the Ombudsman has authority to investigate and prosecute Castro‘s case against respondents in the RTC even as this authority is not exclusive and is shared by him with the regular prosecutors.

LAZATIN V. DESIERTO G.R. 147097 Facts: The Fact Finding Intelligence Bureau of the Office of the Ombudsman filed a Complaint Affidavit against petitioners CARMELO F. LAZATIN, MARINO A. MORALES, TEODORO L. DAVID, and ANGELITO A. PELYAO with Illegal Use of Public Funds. The complaint alleged irregularities in the use of Congressman Lazatin of his Countrywide Development Fund (CDF). Main reason: He was both the proponent and implementer of the

Page 32: 35232938 Crimpro Digest

projects funded by CDF, signed the vouchers and supporting documents pertinent to disbursement and he also was the one who received such. It was alleged that petitioner, together with the help of Morales, David, and Pelayo was able to convert his CDF into cash. After preliminary investigation, Evaluation and Preliminary Investigation Bureau (EPIB) recommended filing 14 counts of Malversation of Public Funds and violation of Section (e) of R.A. No. 3019. Said resolution was approved by Ombudsman, thus 28 Informations were filed against petitioners before the Sandiganbayan. Petitioner submitted reconsideration, it was granted by the Sandiganbayan who ordered reinvestigation. Office of the Special Prosecutor‘s (OSP) recommended dismissal for lack or insufficiency of evidence. The Ombudsman however ordered the Office of the Legal Affairs (OLA) to review the OSP resolution. OLA recommended to proceed with the trial. Ombudsman adopted OLA‘s resolution. Petitioner‘s main allegation is that the Ombudsman had no authority to overturn OSP‘s resolution because of lack of authority to do so. They alleged that base on the constitution, the Ombudsman and OSP are separate entities and thus the prior does not have the power to overturn any decision made the latter. Second, checks were issued to Lazatin as reimbursement for advances he made. Decision of lower courts/CA: Directly filed with SC from Sandiganbayan/Ombudsman. Issue: Whether Ombudsman has the authority to overturn the OSP. Ratio : This Court held that giving the prosecutorial powers to the Ombudsman is in accordance with the Constitution as paragraph 8, 13, Article IX provides that the Ombudsman shall ―exercise with other functions or duties as may be provided by law.‖ Thus, the Legislature passed R.A. No. 6770, which contains Section 3, placing the OSP under the Ombudsman was upheld by this court. Section 7 of Article IX expressly provides that the then existing Tanodbayan, to be henceforth known as the Office of the Special prosecutor, ―shall continue to function and exercise its powers as now or hereafter may be provided by law, except those conferred on the Office of the Ombudsman created under this Constitution.‖ It follows then that Congress may remove any of the Tanodbayan‘s/Special Prosecutor‘s power or grant it other powers, except those powers conferred by the Constitution on the Office of the Ombudsman. Perez v. Sandiganbayan: the power to prosecute carries with it the power to authorize the filinf of informations, which power had not been delegated to the OSP. Decision Ombudsman was properly acting with R.A. No. 6770 and properly exercising its power of control and supervision over the OSP when it disapproved its released resolution Punzalan v. Pena (2004) Topic/Issue: Petition for Certiorari (Rule 1, Sec. 65, Rules of Court) Parties: Petitioners: Rosalinda Punzalan (mother), Randall Punzalan, Rainier Punzalan (shooting victim) Respondents: Dencio Dela Pena (initial victim), Robert Cagara (Plata family driver; gun carrier)

Punzalan & co.: Rainier Punzalan, Randall Punzalan, Ricky Eugenio, Jose Gregorio, Alex ―Toto‖ Ofrin (attempted to stab using balisong), and several others

Dela Pena & co.: Dencio Dela Pena, Michael Plata, Robert Cagara FACTS:

Page 33: 35232938 Crimpro Digest

Alleged crime The Punzalan and Plata families were neighbors in Mandaluyong City. Respondent Dencio Dela Pena was a house boarder of the Plata family. He was standing near the Plata home on the night of 13 August 1997, when the group of Petitioner Rainier Punzalan (referred to as Punzalan & co.) approached him and shouted “Hoy, kalbo, saan mo binili ang sombrero mo?” Dela Pena replied, “Kalbo nga ako, ay pinagtatawan pa ninyo ako.” Irked by Dela Pena‘s response, Punzalan & co. assaulted him by slapping his face, punching and kicking him. Dela Pena escaped just as Toto Ofrin of Punzalan & co. tried to stab Dela Pena with a balisong. Punzalan & co. then chased after Dela Pena. As Dela Pena was fleeing, he encountered Cagara, the Plata‘s family driver. Cagara was carrying a gun, which Dela Pena grabbed and pointed to Punzalan & co. to scare them off. Michael Plata who was nearby, intervened by wrestling the gun away from Dela Pena, which caused it to accidentally fire, hitting Rainier Punzalan on the thigh. Dela Pena, Plata and Cagara (referred to as Dela Pena & co.) ran to the Platas‘ house and locked themselves in. From the outside of the house, Punzalan & co. shouted “Lumabas kayo d’yan, putang ina ninyo! Papatayin naming kayo!” Charges filed Rainier Punzalan filed a criminal complaint against Michael Plata for Attempted Homicide and Robert Cagara for Illegal Possession of Firearm. The Department of Justice (DOJ) directed that Dela Pena be likewise investigated for the charge of Attempted Homicide. Dela Pena & co. then filed countercharges, including Attempted Murder against Punzalan & co., and one for Grave Threats against Toto Ofrin. Subsequently, Cagara filed a criminal complaint for Grave Oral Defamation against Rosalinda Punzalan, mother of Rainier Punzalan. Cagara alleged that during a meeting at the Office of the Prosecutor of Mandaluyong, Rosalinda told him, “Hoy Robert, magkanong ibinigay ng mga Plata sa iyo sa pagtestigo? Dodoblehin ko at ipapasok pa kita ng trabaho.” Rulings of the Prosecutor and Justice Secretary On 28 July 1998, the Assistant City Prosecutor of Mandaluyong issued a Joint Resolution dismissing the complaint against Rosalinda Punzalan and the charge of Attempted Murder filed by Dela Pena against Punzalan & co. On appeal by Dela Pena and Cagara, the Justice Secretary on 23 March 2000 issued a Resolution reversing the Joint Resolution dated 28 July 1998 and modifying the following charges:

1. Grave Oral Defamation against Rosalinda Punzalan – reduced to Slight Oral Defamation

2. Attempted Murder against Punzalan & co. – reduced to Attempted Homicide

3. Grave Threats against Toto Ofrin – reduced to Other Light Threats

On reconsideration, the Justice Secretary on 6 June 2000 set aside the Resolution dated 23 March 2000 and directed the withdrawal of the Informations against Punzalan & co. and Rosalinda Punzalan (i.e. all charges against Petitioners are dropped) Ruling of the Court of Appeals (CA) The Respondents filed a Petition for Certiorari at the CA seeking to reinstate the modified criminal charges as indicated in the ruling of the Justice Secretary dated 23 March 2000. The CA granted their petition, but denied reinstatement of the charge of Other Light Threats against Toto Ofrin. Hence this appeal.

Page 34: 35232938 Crimpro Digest

ISSUES: A. Whether the petition for certiorari was the proper remedy in this case

B. Whether there is sufficient evidence to sustain a finding of probable cause against Petitioners

HELD: A. Whether the petition for certiorari was the proper remedy in this case – YES.

B. Whether there is sufficient evidence to sustain a finding of probable cause against

Petitioners – NO. The Resolution of the Justice Secretary dated 6 June 2000 is

REINSTATED (i.e. all charges against Petitioners are dropped)

RATIO: A. Whether the petition for certiorari was the proper remedy in this case – YES.

1. See relevant Rule: Section 1, Rule 65, Rules of Court: Petition for certiorari.

2. Petition for certiorari is the proper remedy when any tribunal, board or officer exercising

judicial or quasi-judicial functions has acted:

a. without or in excess of its jurisdiction; OR

b. with grave abuse of discretion amounting to lack or excess of jurisdiction; AND

c. there is no appeal, nor any plain, speedy, and adequate remedy at law.

3. Lack of jurisdiction defined – when respondent does not have the legal power to

determine the case.

4. Grave abuse defined – when the respondent, being clothed with the power to determine

the case, oversteps his authority as determined by law. It must amount to:

a. a capricious, whimsical exercise of judgment, OR

b. when the power is exercised in an arbitrary or despotic manner by reason of passion

or personal hostility; AND

c. it must amount to an evasion of positive duty to act in contemplation of law. It is not

enough that there is abuse of discretion, such abuse must be grave.

B. Whether there is sufficient evidence to sustain a finding of probable cause against

Petitioners – NO.

1. Following the definitions above, the Court finds that there was no abuse of discretion

committed by the City Prosecutor:

a. In dismissing the charge of Slight Oral Defamation against Rosalinda Punzalan – the

alleged defamatory statement was uttered within the Office of the City Prosecutor.

The City Prosecutor, being the proper officer at the time of the incident, is the best

person to observe the demeanor and conduct of the parties and witnesses and

determine probable cause. The City Prosecutor, after observing the conduct of the

parties, deemed that there was no probable cause. This was also upheld by the

Justice Secretary in his Resolution. Jurisprudence provides that the question of

whether or not to dismiss a complaint is within the purview of the functions of the

prosecutor, and ultimately, that of the Justice Secretary.

b. In dismissing the charge of Attempted Homicide against Punzalan & co. – this was in

the nature of a countercharge, which the DOJ did not deem necessary to pursue.

After Rainier Punzalan had filed his complaint, the DOJ directed that Dela Pena also

be investigated for the charge of Attempted Homicide filed by Rainier. Therefore the

complaint of Dela Pena of Attempted Homicide against Punzalan & co. was no longer

necessary since the same could be threshed out in proceedings relating to the

Attempted Homicide charge filed by Rainier.

Page 35: 35232938 Crimpro Digest

2. The Court also finds that the Justice Secretary did not commit grave abuse of discretion.

Absent any finding of grave abuse, jurisprudence in Samson, et. al. v. Guingona dictates

that the Court should not interfere in the conduct of preliminary investigations or

reinvestigations. The determination of what constitutes sufficient evidence to establish

probable cause for the filing of an information against an offender must be left with the

investigating prosecutor.

Sesbreňo v. Aglugub Feb. 28, 2005 A.M. No. MTJ-05-1581 Facts: Three complaints were filed in the sala of Judge Aglugub against Enrique Marcelino, Susan Nuňez, Edna Tabazon and Fely Carunungan (who are all members of the traffic unit management group of San Pedro, Laguna). The cases filed were for falsification and usurpation of authority. After conducting a preliminary examination, Aglugub (respondent) issued a consolidated resolution dismissing the cases of falsification and grave threats for lack of probable cause then setting for arraignment the case of usurpation of authority. Upon arraignment, one of the accused, Marcelino did not appear, thus respondent issued a warrant of arrest. Subsequently, complainant Atty. Sesbreňo filed a private complainant‘s urgent manifestation alleging that the accused were also charged of a violation of R.A 10 and praying that warrants also be issued. (take note of this, this is the one relevant to issue related to crimpro.) Acting on this matter, respondent judge issued an order stating that a charge for violation of R.A 10 was already included in the charge of usurpation of authority but was not resolved due to oversight. Respondent also furthered that R.A. 10 applies only to members of seditious organizations engaged in subversive activities , hence there was no probable cause for the complaint. Citing Sec.6 (b), rule 112 of the revised rules of criminal procedure, respondent judge dismissed the case and forwarded the records to the Provincial Prosecutor‘s Office (PPO) for review. (fact relevant to topic) Complainant filed a Motion for reconsideration and Urgent Ex-Parte Motion for issuance of Warrant of Arrest Against non- appearing accused. Respondent judge did not act on these arguing that the court already lost jurisdiction on over the case. (because it was already forwarded to the PPO) During the hearing, accused Tabazon, Carunungan and Nuňez did not appear, but petitioner did not mover for the issuance of warrants nor the cancellation of the hearing, instead he filed a complaint against the judge, charging him with Gross ignorance of the law, neglect of duty and conduct prejudicial to the best interest of the service relative to criminal case of People v. Marcelino et al, hence this case. ISSUES: W/N the respondent judge erred in conducting a preliminary investigation for the charge of usurpation of authority W/N respondent judge erred in not issuing warrants of arrest for failure of the accused to appear during trial W/N respondent judge erred in dismissing the complaint for violation of R.A. 10 W/N respondent judge erred in transmitting the case to the PPO instead of the ombudsman. (only this issue will be discussed because the case is under the topic of ombudsman) Arguments: Respondent argues that R.A 6770, (ombudsman act of 1989), the PPO has been designated as the Deputized Ombudsman Prosecutor, thus the PPO can take action on similar cases for review and appropriate action. Complainant argues further that there is no provision in the Ombudsman Act of 1989 specifically deputizing the PPO to be the ―deputized ombudsman prosecutor‖ and that respondent failed to

Page 36: 35232938 Crimpro Digest

comply with Administrative order no. 8 since he has yet forwarded her resolution to the Deputy Ombudsman. Decision: YES. When respondent judge forwarded the case to the PPO she based her action on Sec. 5 of Rule 112 which provides that the resolution of the investigating judge is subject to review by the provincial or city prosecutor, or the ombudsman or deputy as the case may be and that under RA 6770, the Provincial prosecutor has jurisdiction over the case. (violation of R.A10). Although this may be true, Sec. 31 of R.A 6770 provide3s that prosecutors can (be) deputized by the ombudsman to act as special investigator or prosecutor only in certain cases. Such provision in not applicable to this case. ESQUIVEL vs OMBUDSMAN Facts PO2 Eduardo and SPO1 Catacutan are both residents of Nueva Ecija but assigned with the Regional Intelligence Division in Pampanga. Esquivel is the elected municipal mayor of Jaen, Nueva Ecija. They charged petitioner who is a municipal mayor of Jaen, Nueva Ecijia and his brother who is barangay captain for illegal arrest, arbitrary detention, maltreatment, attempted murder and grave threats. PO2 Eduardo was about to eat lunch at his parent's house in Nueva Ecija when petitioners arrived and disarmed Eduardo of his pistol. They forced him to board petitioner's vehicle and brought him to the Municipal Hall. The mayor believed that Eduardo was among those who raided his jueteng den and arrested members of crime syndicates connected with him. PO2 Eduardo also stated that while their way to the town hall, the Mayor hauled him with the use of a firearm and threatened to kill him. The mayor then struck him with a handgun which caused him to lose consciousness. When he awoke, he was forced to sign a statement in the police blotter that he was in good physical condition so that he could be released. The records of Eduardo's complaint was forwarded to the Office of the Deputy Ombudsman for Luzon for appropriate action. The Office of the Ombudsman conducted a preliminary investigation and required the petitioners to file their counter affidavits. They stated that Eduardo is a fugitive from justice and was subject to an illegal possession of firearm complaint. The deputy ombudsman then recommended that the Mayor be indicted for the crime of grave threats only and his brother with less serious physical injuries. The other charges against PO2 were dismissed. The Ombudsman approved the recommendation/resolution. thereafter, information was filed against petitioners with the Sandiganbayan. Petitioners moved for reconsideration of the resolution of the Deputy Ombudsman. The motion was then denied by the OSP. Petitioners were then arraigned and pleaded not guilty to the charges. They then claimed grave abuse of discretion on the part of public respondents in rendering the resolution with the Supreme Court. Issue 1. w/n the Ombudsman committed grave abuse of discretion in directing the filing of the information against petitioners 2. w/n the Sandiganbayan committed grave abuse of discretion in assuming jurisdiction over the criminal cases. Held 1. No. 2. No. Decision DISMISSED.The petition is without merit. Petitioner's Argument 1 The Ombudsman committed grave abuse of discretion when he failed to consider the evidence that PO2 Eduardo was in good physical condition.

Page 37: 35232938 Crimpro Digest

Ratio 1 The Ombudsman is empowered to determine whether there exists reasonable ground to believe that a crime has been committed and that the accused is probably guilty thereof and thereafter file the corresponding information with the appropriate courts. The Supreme Court will not ordinarily interfere with the Ombudsman's exercise of his investigatory and prosecutory powers without good and compelling reason or absent a showing of grave abuse of discretion. Petitioners failed to prove such. There was no abuse of discretion on the Ombudsman in disregarding the admission found in the document signed by PO2 stating tthat he was in good physical condition. The admission merely applied to the execution of the document and not the truthfulness of its content. Petitioner's Argument 2 The Sandiganbayan has no jurisdiction over their persons as they hold positions excluded in RA 7975. As the positions of municipal mayor and barangay captain were not mentioned, they claimed that they are not covered. Ratio 2 Municipal mayor fall under the original and exclusive jurisdiction of the Sandiganbayan. it is only in cases where none of the accused are occupying positions corresponding to salary grade 27 or higher that exclusive original jurisdiction shall be vested in the proper RTC, MTC etc. Under the Local Government Code, the muncipal mayor has a salary grade of 27. Since his brother is a co-accused with the mayor who is covered by the RA, he is likewise covered though he may be only a barangay captain. Sandiganbayan's jurisdiction over the criminal cases is clearly founded on law. G.R. No. 167764 October 9, 2009 Vicente Foz, Jr. and Danny G. Fajardo, Petitioners, Vs. People of the Philippines, Respondent. Facts: Petitioners are accused of libel in this case because of an article printed in a daily publication called Daily Panay. The article was supposedly libelous because it attacked a certain Dr. Portigo, exposing him to ―public hatred, contempt and ridicule‖. The article spoke of Dr. Portigo being an incompetent physician, taking advantage of the poor for his monetary gain. The article was published July 5, 1994. On March 1, 1995, petitioners pleaded not guilty; thereafter trial ensued. On December 4, 1997, the RTC of Iloilo finds petitioners guilty beyond reasonable doubt and sentenced them to 3 months 11 days of arresto mayor as minimum to 1 year eight months 21 days of prision correcional as maximum and a fine of P1,000 each. Petitioners filed a motion for reconsideration on February 20, 1998 but it was denied. Petitioners then filed an appeal to the CA; on November 24, 2004, the CA affirmed in toto the RTC decision. The petitioners then filed a motion for reconsideration but was denied in a resolution dated April 8, 2005. Petitioners then elevated the case to the Supreme Court, raising for the first time the issue of whether or not the RTC of Iloilo City Branch 23 had jurisdiction over the case. The Supreme Court said that the issue of jurisdiction can be raised anytime, even on appeal or motions for reconsiderations, which leaves the case to one final issue. Issue: Whether or not the RTC of Iloilo City, Branch 23 had jurisdiction to try the case

Page 38: 35232938 Crimpro Digest

Ruling: No, the RTC of Iloilo. Branch 23 had no jurisdiction Ratio: Libel cases can only be instituted in 4 places according to the rules on venue in Article 360.

1. Whether the offended party is a public official is a public official or private person, the criminal action may be filed in the CFI of the province or city where the libelous article is printed and first published.

2. If the offended party is a private individual, the criminal action may also be filed in the CFI of the province where he actually resided at the time of the commission of the offense.

3. if the offended party is a public officer whose office is in Manila at the time of the commission of the offense, the action may be filed in the CFI of Manila.

4. If the offended party is a public officer holding office outside of Manila, the action may be filed in the CFI of the province where he hel d office at the time of the commission of the offense.

In the present case, Dr. Portigo is a private individual at the time of the publication of the offense, so the venue is either the place where the article was first published or the place of his residence. In the information filed, it only alleged that Daily Panay was ―in general circulation in the province of Iloilo‖, nowhere is there proof that the article was indeed first published in the city of Iloilo. In the case of Dr. Portigo‘s residence, the information only alleged that ―Dr. Edgar Portigo is a physician and medical practitioner in Iloilo City.‖ According to the SC, this does nothing to prove that Dr. Portigo is actually residing in Iloilo City, the only thing it stresses was that Dr. Portigo was a medical practioner in Iloilo City. Finally the Supreme Court stresses the importance of the fact that jurisdiction of a court over a criminal case is determined by the allegations of the information, and based on the information, there was no clear fact indicating that the RTC of Iloilo City, Branch 23 had jurisdiction over the case, therefore the petition was granted and the criminal case was dismissed.

BERNADETTE MONDEJAR v. JUDGE MARINO S. BUBAN A.M. No. MTJ-01-1349. July 12, 2001

FACTS:

In a sworn letter complaint Bernadette Mondejar charged Judge Marino S. Buban, MTCC Tacloban City, with gross ignorance of the law, partiality, serious irregularity and grave misconduct relative to a Criminal Case for violation of BP 22.

Mondejar alleged that Judge Buban issued a ―hold departure order‖ against her on October 23, 1998 in violation of SC Circular No. 39-97 which provides that “hold departure orders” shall be issued only in criminal cases within the exclusive jurisdiction of the RTC. She further alleged that respondent judge did not give her an opportunity to be heard before issuing the questioned order.

Judge Buban admitted having issued the hold departure order because he was not aware of the Circular. He was not furnished a copy and managed to secure a copy only after he instructed his legal researcher to get one from the Executive Judge of the Tacloban RTC. Later on he issued an order lifting and setting aside the hold departure order. As regards the issue of denial of due process, he said that Mondejar and counsel were duly notified of the scheduled hearing but neither appeared on said date.

Page 39: 35232938 Crimpro Digest

ISSUE: W/N Judge has jurisdiction to issue the hold departure order. HELD: NO. RATIO:

Circular 39-97 limits the authority to issue hold-departure orders to criminal cases within the jurisdiction of second level courts. Par. 1 of the circular specifically provides that ―hold-departure orders shall be issued only in criminal cases within the exclusive jurisdiction of the regional trial courts.‖ Clearly then, criminal cases within the exclusive jurisdiction of first level courts do not fall within the ambit of the circular, and it was an error on the part of respondent judge to have issued one in the instant case.

Canon 3, Rule 3.01 of the Code of Judicial Conduct exhorts judges to be ―faithful to the law and maintain professional competence.‖ Members of the bench must exert due diligence in keeping abreast with the development in law and jurisprudence. Judges should always be vigilant in their quest for new developments in the law so they could discharge their duties and functions with zeal and fervor. Also, Circular 39-97 is not a new circular.

People vs. Sandiganbayan Facts:

Jose S. Ramiscal, Jr., Julian Alzaga, Manuel Satuito, Elizabeth Liang and Jesus Garcia were all charged with Malversation through Falsification of Public Documents before the Sandiganbayan. They apparently misappropriated for their personal use P250,318,200.00 from the funds of the Armed Forces of the Philippines Retirement and Separation Benefits System (AFP-RSBS).

Ramiscal filed with the Sandiganbayan an "Urgent Motion to Declare Nullity of Information and to Defer Issuance of Warrant of Arrest."

2 He argued that the Sandiganbayan had

no jurisdiction over the case because the AFP-RSBS is a private entity. The Urgent Motion was denied by the Sandiganbayan.

Respondents filed a Motion for Reconsideration. The Sandiganbayan this time sustained respondents' contention that the AFP-RSBS is a private entity. Hence, it reconsidered its earlier Resolution and ordered the dismissal of Criminal Case No. 25741 for lack of jurisdiction.

The prosecution filed for a Motion for Reconsideration, which was denied. Thus, the instant special civil action for certiorari. Issues: a) W/N Sandiganbayan grave abuse of discretion in dismissing the case for lack of jurisidiction b) W/N Sandiganbayan committed grave abuse of discretion in denying prosecution‘s motions for reconsideration

Ratio: a) YES. The AFP-RSBS was created by Presidential Decree No. 361. Its purpose and functions are akin to those of the GSIS and the SSS, as in fact it is the system that manages the retirement and pension funds of those in the military service. While it may be true that there have been no appropriations for the contribution of funds to the AFP-RSBS, the Government is not precluded from later on adding to the funds in order to provide additional benefits to the men in uniform. The above considerations indicate that the AFP-RSBS is imbued with public interest. As such, it is a government entity. Sandiganbayan has jurisdiction over the case at hand. b) The resolution of the Sandiganbayan was a final order that finally disposed of the case. Thus, Rule 45 of the 1997 Rules of procedure, and Section 7 of PD1606 apply. These state that:

Decisions and final orders of the Sandiganbayan shall be appealable to the Supreme Court by petition for review on certiorari raising pure questions of law in accordance with Rule 45 of the Rules of Court. Basic is the rule that a special civil action for certiorari under Rule 65 of the Rules may be availed of only where there is no appeal, or any plain, speedy, and adequate remedy in

Page 40: 35232938 Crimpro Digest

the ordinary course of law.6 Certiorari cannot be availed of as a substitute for the lost

remedy of an ordinary appeal. Said rule may be relaxed when the issue raised is one purely of law, where public interest

is involved, and in case of urgency. Certiorari may also be availed of where an appeal would be slow, inadequate and insufficient, and where strict application of the rules would frustrate the ends of justice. PANAGUITON V DOJ GR 167571 FACTS Cawili and his business associate Tongson borrowed from Panaguiton (petitioner) sums amounting to 1,979,459. Cawili and Tongson issued checks signed by both of them to Panaguiton but these were dishonored upon presentation. Panaguiton made demands to pay but to no avail. He formally filed a complaint on August 24 1995 for violating BP 22 before the City Prosecutor‘s Office. Tongson moved to drop his name from the case as his signatures were allegedly falsified. Case against him was dismissed but afterwards upon finding that Tongson might have indeed signed the checks, the chief state prosecutor directed the city prosecutor to conduct a reinvestigation. Tongson moved for reconsideration but denied. In 1999 assistant prosecutor dismissed the complaint for the action has prescribed pursuant to Act 3326, which provides for the prescriptive periods of statutes without their own (4 years for BP22). She claims that the filing of the complaint on August 24 1995 did not interrupt the running of the period as the law refers to judicial and not administrative proceedings. Panaguiton appealed to the DOJ, but was dismissed. He moved for reconsideration two times before it was held that the prescription had not elapsed. However in another resolution the DOJ ruled that prescription has set in. Panaguiton filed certiorari petition in CA but was dismissed for failure to attach a proper verification and certification for non-forum shopping and that the DOJ resolution (saying prescription has set in) dated August 9 2003 submitted was a mere photocopy. Panaguiton complied but the CA still denied. ISSUE Procedural: P.1. W/N verification subsequently attached by Panaguiton substantially complies with the rules P.2. W/N dismissal of the case for failure to attach certified true copy was warranted (he submitted only a photocopy) Substantive: S.1. W/N The filing of the complaint in the prosecutor‘s office tolled the prescriptive period. HELD P.1. YES. Subsequent attachment substantially complied with requirements of the law. P.2. NO. Case shouldn‘t have been dismissed. S.1. YES. Filing of the complaint in the prosecutor‘s office tolls the prescriptive period for violations of BP22. RATIO P.1. The verification was intended simply to secure an assurance that the allegations therein are true and not a mere product of fiction. Deficiency in the verification may be excused or dispensed

Page 41: 35232938 Crimpro Digest

with as this doesn‘t affect jurisdiction. At times the court even waives the requirement or just allows corrections. P.2. A certified true copy was actually attached in Annex A for the August 9 resolution. The photocopy attached was for the August 3 resolution and this wasn‘t the resolution the petitioner seeks to be reversed. S.1. When Act 3326 was passed into law, preliminary investigation of cases was done by the justices of peace, and not by agents of the executive department (i.e. prosecutors). Thus, the prevailing rule at that time is that prescription is tolled once filed with the justice of peace (a judicial process). However, since then, the conduction of a preliminary investigation was moved to the function of the executive department. Today, the term proceedings must be understood to mean either executive or judicial proceedings. With this interpretation, any type of investigation may ultimately lead to sufficiently toll prescription. To rule otherwise would deprive the injured party the right to obtain vindication on account of delays not under his control. As seen in this case, various conflicting opinions of the DOJ delayed his cause. Aggrieved parties who do not sleep on their right should not be allowed to suffer simply because of circumstances beyond their control.